General Medicine (No=417)
In Fabry's disease, the leading cause of death is:
renal failure
colorectal cancer
cerebrovascular accident
myocardial infarction
Your answer was INCORRECT
Explanation
In Fabry's disease, deficiency of alpha galactosidase A causes ceramide accumulation in blood
vessels leading to diffuse angiopathy, with myocardial infarction or stroke quite common. Still,
renal failure is the leading cause of death.
Ocular features of Fabry's disease include: vortex keratopathy, conjunctival aneurysms, posterior
spoke-like cataracts, papilloedema, macular oedema, optic atrophy and retinal vascular dilation.
All of the following conditions are known to be associated with keratoconus EXCEPT:
Type 1 diabetes mellitus
Vernal keratoconjunctivitis
Atopic dermatitis
Ehlers-Danlos syndrome
Down's syndrome
Your answer was CORRECT
Explanation
Diabetes is not associated with keratoconus. Other options supplied are true.
Which one of the following concerning Reiter's syndrome is FALSE?
most patients are HLA-B27-positive
90% of cases occur in men
the most common ocular finding is an acute non-granulomatous anterior uveitis
it may follow a bout of either urethritis or dysentery
Your answer was CORRECT
Explanation
The most common eye finding in Reiter's syndrome is a non-specific conjunctivitis. Non-
granulomatous iritis, which can be bilateral and chronic, is less common. Keratoderma
blennorrhagicum (a rash similar to pustular psoriasis) and circinate balanitis are also diagnostic
criteria for the disease.
All of the following cause granulomatous inflammation EXCEPT:
Treponema pallidum
mycobacterium
Moraxella
Bartonella
Borrelia bergdorferi
Your answer was INCORRECT
Explanation
Granulomatous reactions are caused by intra-cellular organisms, which initiate a cell mediated
response with marcrophages, lymphocytes and plasma cells. Moraxella is a Gram negative bacilli
and would not typically cause a granulomatous response.
Which condition involves deposition of a light-chain-derived amyloid:
familial Mediterranean fever
Waldenstrom macroglobulinaemia
rheumatoid arthritis
Alzheimer's disease
Your answer was CORRECT
Explanation
Amyloid can be derived from proliferation of light chain fragments of immunoglobulin, as in
Waldenstrom macroglobulinaemia and multiple myeloma.
A 25-year-old woman presents with primary hyperparathyroidism. It emerges that she has been
treated previously for a prolactinoma.
What is your diagnosis?
Polyglandular autoimmune syndrome type 1
Familial parathyroid hyperplasia
Multiple endocrine neoplasia type 1
Neurofibromatosis type 1
von Hippel-Lindau
Your answer was CORRECT
Explanation
The history of primary hyperparathyroidism and a pituitary tumour in this patient is diagnostic of
multiple endocrine neoplasia type 1 (MEN1). This is an autosomal dominant disorder that is
caused by mutations in the MEN1 gene on 11q13. It is characterised by tumours of the parathyroid
glands, pituitary and pancreatic islet cells. About 90% of patients develop parathyroid hyperplasia
or adenoma. Pituitary tumours are seen in about 60% of patients. They include prolactinomas,
growth hormone-producing tumours and ACTH-producing tumours. Between 50 to 75% of
patients with MEN1 develop pancreatic islet cell tumours that include gastrinoma, insulinoma,
glucagonoma and VIPoma.
Most patients with MEN1 present with hypercalcaemia due to primary hyperparathyroidism.
Presenting features of pituitary tumours can include galactorrhoea and infertility (due to
prolactinoma), acromegaly or Cushing's disease. Pancreatic islet cell tumours can present with
Zollinger–Ellison syndrome (due to gastrinoma), watery diarrhoea with hypokalaemia and
metabolic acidosis or Verner–Morrison syndrome (due to VIPoma), recurrent hypoglycaemia (due
to insulinoma), and hyperglycaemia and necrolytic migratory erythema (due to glucagonoma).
The MEN1 gene encodes a tumour suppressor protein called MENIN.
A patient with Behcet's disease is commenced on a steroid-sparing agent. He subsequently
develops gingival hyperplasia, raised blood pressure and derangement of liver and kidney
function.
Which agent was he most likely taking?
methotrexate
tacrolimus
azathioprine
ciclosporin
Your answer was INCORRECT
Explanation
This patient's side-effects fit with the use of ciclosporin, which can cause:
nephrotoxicity
hepatotoxicity
hypertension
hyperlipidaemia
gingival hyperplasia
hirsutism
Helpful laboratory tests in the evaluation of a patient with suspected sarcoidosis include:
1. angiotensin-converting enzyme (ACE)
2. serum lysozyme
3. chest x-ray
4. antineutrophil cytoplasmic antibody assay (ANCA)
1 and 3
2 and 4
1, 2, and 3
4 only
1, 2, 3, and 4
Your answer was CORRECT
Explanation
An elevated angiotension-converting enzyme (ACE) level occurs in approximately two thirds of
patients with sarcoidosis, and an abnormal chest x-ray (hilar and/or mediastinal adenopathy) is
very likely to be found. Elevated serum lysozyme is more sensitive but less specific. Gallium scan
also may be helpful in the diagnosis. The antineutrophil cytoplasmic antibody (ANCA) assay is
useful in diagnosing Wegener's granulomatosis, not sarcoidosis.
A 38-year-old female presents with a painful right eye. On examination, you note interpalpebral
peripheral corneal stromal thinning with an epithelial defect. You also note a butterfly rash and
blood results show ANA is positive.
What is the most likely diagnosis?
Systemic lupus
Rheumatoid arthritis
Sarcoidosis
Wegener's granulomatosis
Your answer was CORRECT
Explanation
All choices above may cause peripheral ulcerative keratitis. SLE is the most likely in a middle-
aged female, with a butterfly facial rash and ANA positivity.
A patient with intermediate and posterior uveitis has a recent travel history of trekking in the
United States. He describes having a circinate rash on his leg at the time.
What is the most likely cause?
Bartonella henselae
Treponema pallidum
Borrelia burgdefori
Histoplasmosis
Your answer was INCORRECT
Explanation
Lyme disease is caused by the spirochaete Borrelia burgdefori. It is transmitted by a bite from the
Ixodes deer tick, which is endemic to the North East and upper mid-West of the United States.
Early symptoms may include fever, headache and fatigue. Conjunctivitis may occur at this early
stage (Stage I). A rash occurs in 70-80% of infected persons at the site of the tick bite after a delay
of 3-30 days and may or may not appear as the well-publicized bull's-eye rash called erythema
migrans. Later symptoms (weeks to months later, Stage II) include CNS, cardiac and neurological
symptoms, and ocular features include uveitis and retinitis.
This question came in the 2014 FRCOphth.
What is the most common ocular manifestation during Stage 1 Lyme disease?
oculomotor palsies
conjunctivitis
intermediate uveitis
anterior uveitis
Your answer was CORRECT
Explanation
The stages of Lyme disease are as follows:
Stage 1: is characterised by a pathognomic bull's-eye skin rash called erythema migrans and
occurs 3-30 days following a bite from the Ixodes tick. The most common ocular
manifestations during this stage of disease are conjunctivitis and photophobia, which are
usually mild and ophthalmologists are rarely consulted at this point.
Stage 2: occurs weeks to months later and may include neurologic, cardiac and arthritic
complications. It is during this stage that significant ocular manifestations may develop. The
most common presentations are various neuro-ophthalmic signs including oculomotor palsies
and optic nerve involvement.
Stage 3: it is during this stage that most severe ocular complications develop. These include
episcleritis, sympblepharon, keratitis, iritis, posterior or intermediate uveitis, pars planitis,
vitritis, chorioretinitis, exudative retinal detachment, retinal pigment epithelial detachment,
CMO, branch retinal artery occlusion, retinal vasculitis, orbital myositis, and cranial nerve
palsies. Of this group, keratitis, vitritis and pars planitis are the most common.
Infection with which pathogen is characteristically associated with an owl-eye inclusion body:
Chlamydia trachomatis
herpes zoster virus
cytomegalovirus
Borrelia bergdoferi
Your answer was CORRECT
Explanation
Owl-eye inclusion bodies are characteristic of cytomegalovirus infection as seen for example in
cytomegalovirus retinitis in immunocompromised patients.
Which enzyme test should be ordered alongside serum ACE in order to increase the sensitivity
and specificity of diagnosing sarcoid?
amylase
alkaline phosphatase
alanine transaminase
lysozyme
Your answer was CORRECT
Explanation
Lysozyme has good sensitivity but less specificity than serum ACE in the diagnosis of sarcoid.
Using both tests increases the sensitivity and specificity of diagnosing sarcoid.
Which is FALSE regarding Kearns-Sayre syndrome?
elevated CSF protein level may predict patients at risk of developing heart block
the pigmentary retinopathy is generally severe
heart block develops late in the course of the syndrome
disease onset is before 20 years of age
Your answer was INCORRECT
Explanation
Pigmentary retinopathy in Kearns-Sayre syndrome is typically mild. Other statements are true.
Kearns-Sayre syndrome is characterised by:
mitochondrial inheritence
ragged red fibres on muscle biopsy
presentation in 1st or 2nd decades
ptosis
external ophthalmoplegia
cardiac conduction defects
deafness
diabetes
short stature
mild pigmentary retinopathy
All are true of Toxoplasma EXCEPT:
adult chorioretinal involvement implies secondary reactivation
it is an obligate intracellular protozoon
infection can be acquired from inadequately cooked meat
congenital toxoplasma may result in intracranial calcification and mental retardation
Your answer was CORRECT
Explanation
Adult disease may be from reactivation of prior infection in utero or it may arise from primary
infection in adulthood.
What is the probability, given the HLA-B27 genotype, of sacroiliac disease?
<5%
50%
75%
25%
Your answer was CORRECT
Explanation
Up to 25% of individuals with HLA-B27 develop sacroiliac disease. Symptoms of sacroiliac
disease may be subtle. Personal or family history of back problems in patients with iritis should
prompt the physician to obtain sacroiliac radiographs.
Which of the following statements about Marfan's syndrome is FALSE?
most cases are autosomal dominant
typical refraction is moderate hyperopia
both aortic and mitral valve insufficiency are associated
most patients develop ectopia lentis
Your answer was CORRECT
Explanation
Autosomal-dominant patterns are most frequently reported for Marfan's, although 15% of patients
have no family history. Up to 60% of patients with rigorously diagnosed Marfan's syndrome will
develop ectopia lentis. Myopia, high astigmatism and frank keratoconus are other ocular
associations.
Features of Marfan's:
AD inheritance
normal intelligence
arachnodactyly
increased arm span:height ratio
high arched palate
cardiac valvular anomalies (both aortic and mitral)
aortic root aneurysm
Ocular associations
o ectopia lentis superotemporal (zonules present, intact accommodation)
o myopia
o glaucoma
o lattice degeneration
o retinal detachment
o astigmatism
o keratoconus
o hypoplasia of dilator pupillae (miosis)
o rarely flat cornea and blue sclera
Which of the following is most likely to cause erythema multiforme:
penicillin
herpes simplex
adenovirus
mycoplasma
sulfonamides
Your answer was CORRECT
Explanation
Figure: Stevens-Johnson syndrome
Erythema multiforme is a hypersensitivity reaction usually triggered by infections, most
commonly herpes simplex virus (HSV). The next most common cause is Mycoplasma
pneumoniae. Other infective causes include: HZV, adenovirus, hepatitis viruses, HIV and CMV.
Drugs are a very rare cause of erythema multiforme, and if drugs are considered the underlying
aetiology then other reactions such as Stevens-Johnson syndrome and toxic epidermal necrolysis
need to be considered.
Clinically, erythema multiforme presents with a skin eruption characterised by a typical target
lesion. There may be mucous membrane involvement. It is acute and self-limiting, usually
resolving without complications.
Erythema multiforme is divided into major and minor forms. Erythema multiforme major was
once considered part of Stevens-Johnson syndrome but it is now regarded as distinct from
Stevens–Johnson syndrome (SJS) and toxic epidermal necrolysis (TEN).
Stevens-Johnson syndrome (erythema multiforme major) can be caused by:
drugs: sulphonamides, penicillin, phenytoin
infection: HSV, HIV, adenovirus, mycoplasma
malignancy
A 4-year-old child develops gradual onset of progressive personality change and psychomotor
deterioration with myoclonus and seizures. Ocular examination reveals posterior uveitis bilaterally
with papillitis and macular oedema.
What is the most likely causative organism?
rubella
measles
mumps
syphillis
Your answer was INCORRECT
Explanation
This child has symptoms of subacute sclerosing panencephalitis. This is a chronic progressive
neurodegenerative disease of children caused by the measles virus. Ocular findings include
posterior uveitis with papillitis, macular oedema, whitish retinal infiltrates and choroiditis.
A 40-year-old Japanese man presents with apthous mouth ulcers, arthritis, bilateral mobile
hypopyon, and occlusive retinal arteritis.
The disorder may also demonstrate which one of the following systemic features?
Kyphoscoliosis
Pulmonary artery aneurysm
Hilar adenopathy
Urethritis
Your answer was INCORRECT
Explanation
Behcet's disease may be associated with painful hypopyon iridocyclitis. The eye can also be
remarkably quiet in the presence of a hypopyon in Behcet's disease, unlike in HLA-B27-
associated disease, in which pain and photophobia dominate as symptoms. A strong association
with HLA-B1/HLA-B51 is often present in Behcet's disease.
Systemic features of Behcet's include:
acne
erythema nodosum
aphthous ulcers
genital ulcers
interstitial lung changes
pulmonary artery aneurysm (pathognomonic finding)
Treatment is with systemic and periocular corticosteroids. Colchicine may be useful in reducing
recurrences of the disease. Cyclosporine and cytotoxic agents may also be required
A 60-year-old Afro-Caribbean patient complains of a whitish area on her left eye for several
weeks. Her eye has been slightly uncomfortable but with no discharge. On examination, there is
peripheral corneal stromal thinning temporally with fluorescein pooling and corneal
neovascularisation.
Which blood test is LEAST helpful in investigating the underlying diagnosis?
anti-neutrophil cytoplasmic antibody
rheumatoid factor
purified protein derivative skin test
HIV antibodies
Your answer was CORRECT
Explanation
This patient has signs suggestive of peripheral ulcerative keratitis.
Underlying systemic causes of PUK include:
rheumatoid arthritis
Wegener's granulomatosis
polyarteritis nodosa
relapsing polychondritis
infection: TB, syphilis
HIV or AIDS are not recognised causes of PUK.
A patient presents with band keratopathy, conjunctival calcification and corneal opacification.
From the patient's medical history, you suspect these features are related to his background
endocrine abnormality.
Which condition does he most likely have?
hypoparathyroidism
hyperthyroidism
Addison's disease
hypothyroidism
hyperparathyroidism
Your answer was CORRECT
Explanation
Hyperparathyroidism is usually caused by a solitary parathyroid adenoma. Features include:
systemic
o fatigue
o renal stones
o polyuria
o bone pain
o peptic ulcers
ocular
o band keratopathy
o conjunctival calcification
o corneal opacification
You see a healthy 20-year-old man in your clinic. There is no significant past medical history and
he plays rugby for Saracens rugby club. He tells you that his brother died at the age of seven years
with cystic fibrosis.
What are his chances of being a carrier?
1 in 4 (25%)
2 in 3 (67%)
1 in 2 (50%)
1 in 3 (33%)
3 in 4 (75%)
Your answer was INCORRECT
Explanation
Cystic fibrosis (CF) is an autosomal recessive disorder that is caused by mutations in the cystic
fibrosis transmembrane conductance regulator (CFTR) gene on 7q31.2. Affected individuals have
mutations in both alleles of this gene.
The siblings of an affected individual have:
a 1 in 4 (25%) chance of being affected with this condition
a 1 in 2 (50%) chance of being CF carriers
a 1 in 4 (25%) chance of being neither affected nor a carrier
Note that the proband in this question is a healthy 20-year-old man and he is therefore unaffected
with CF. When affected siblings are removed from the above equation then the unaffected siblings
have a 2 in 3 (67%) chance of being CF carriers and 1 in 3 (33%) chance of not being carriers.
Which condition is NOT associated with peripheral ulcerative keratitis?
relapsing polychondritis
rheumatoid arthritis
Wegener granulomatosis
primary Sjogren syndrome
Your answer was CORRECT
Explanation
Figure: Peripheral ulcerative keratitis
Primary Sjogren syndrome affects the salivary and parotid glands (causing dry mouth) and the
lacrimal glands (causing dry eye). It is not associated with peripheral ulcerative keratitis.
Common systemic causes of peripheral ulcerative keratitis:
rheumatoid arthritis
Wegener granulomatosis
relapsing polychondritis
systemic lupus erythematosus
All of the following are true regarding juvenile idiopathic arthritis EXCEPT:
pauci-articular JIA is associated with increased risk of uveitis
ocular involvement usually occurs after joint involvement
it is an inflammatory arthritis of greater than 6 weeks duration before age 18
early onset of JIA is associated with increased risk of uveitis
Your answer was INCORRECT
Explanation
JIA is an inflammatory arthritis of greater than 6 weeks duration before age 16. Other options
supplied are true.
Risk factors for ocular involvement in JIA include:
ANA positive
rheumatoid factor negative
pauciarticular type (90% of cases)
female
involvement of lower extremity joints
lack of involvement of the wrists
Which statement about thrombus is FALSE:
platelets form the first layer of thrombus
it can cause embolism
it involves mainly the intrinsic and extrinsic clotting cascade
it can occur in both arteries and veins
Your answer was INCORRECT
Explanation
A thrombus is a solid mass of blood constituents forming within the vascular system, i.e. both
arteries and veins. Unlike simple clotting in which the clotting cascade plays an important role,
thrombus is formed by the interaction between the platelets and the blood constituents. Fragments
of the thrombus may break off into circulation causing embolism.
All of the following cutaneous manifestations may be seen in Vogt-Koyanagi-Harada syndrome
EXCEPT:
alopecia
folliculitis
vitiligo
poliosis
Your answer was CORRECT
Explanation
Cutaneous manifestations of Vogt-Koyanagi-Harada include:
alopecia
vitiligo
poliosis
These manifestations confirm the notion that melanin is the immunogenic agent that results in the
intraocular inflammation in this condition.
A positive pathergy test occurs when the skin is pricked with a needle and:
there is pustule formation
there is induration greater than 5mm diameter
there is wheal formation
there is induration greater than 2 mm diameter
Your answer was CORRECT
Explanation
A positive pathergy test is the formation of a pustule following pricking of the skin with a sterile
hypodermic needle.
All of the following can be teratogens in pregnancy EXCEPT:
retinoic acid
alcohol
folic acid
warfarin
sodium valproate
Your answer was CORRECT
Explanation
Teratogens include:
Sodium valproate: neural tube defects
Retinoic acid: cranio-facial, brain and limb defects
Warfarin: interferes with cartilage development, hypoplastic nose and chondrodysplasia
Alcohol: growth retardation, microcephaly, short palpebral fissures and smooth philtrum as
part of the fetal alcohol syndrome
Folic acid (400 mcg/day) is recommended before and through the first few months of pregnancy
in the prevention of neural tube defects.
Ocular manifestations of psoriatic arthritis include all EXCEPT:
intermediate uveitis
marginal keratitis
anterior uveitis
Sjogren's syndrome
Your answer was CORRECT
Explanation
Ocular manifestations of psoriatic arthritis include:
anterior uveitis
marginal keratitis
conjunctivitis
Sjogren's syndrome
Which medication is MOST likely to cause hypokalaemia?
amiloride
ramipril
spironolactone
bendroflumethiazide
Your answer was CORRECT
Explanation
Bendroflumethiazide is a thiazide diuretic which can be associated with increased loss of
postassium leading to hypokalaemia. Amiloride and spironolactone are potassium-sparing
diuretics, so are not likely to cause hypokalaemia. As an ACE inhibitor, ramipril causes
hyperkalaemia not hypokalaemia
An 83-year-old man has a resting tremor, cog-wheel rigidity and bradykinesia.
What is the most likely diagnosis?
Motor neuron disease
Friedrich's ataxia
Wilson's disease
Meige's syndrome
Parkinson's disease
Your answer was CORRECT
Explanation
Features described are typical of Parkinsonism.
A 55-year-old Caucasian lady, who is systemically well, presents with right-sided photophobia
and mildly reduced vision of 2 weeks duration. On examination, there is right granulomatous
anterior uveitis, with an IOP of 32mmHg. There are patches of iris atrophy. There is no posterior
segment inflammation, mild punctate epithelial staining of the cornea and no rash.
Clinical features are most suggestive of which diagnosis:
ankylosing spondylitis-associated uveitis
Behcet's uveitis
herpes simplex uveitis
toxoplasmosis
sarcoidosis
Your answer was CORRECT
Explanation
The findings suggest herpes simplex uveitis, which may occur in the absence of a typical herpetic
epithelial defect. Raised IOP is a common feature due to trabeculitis. Iris atrophy is also
commonly observed due to iris vasculitis. Corneal sensation would be useful to perform in this
case.
Ankylosing spondylitis is unlikely because it typically affects males who are HLA-B27 positive
and is typically non-granulomatous.
Behcet's disease is unlikely because it is typically non-granulomatous, often bilateral and posterior
segment findings such as peri-phlebitis and occlusive vasculitis usually predominate.
Sarcoid is a possibility as it can cause a granulomatous uveitis, however granulomatous uveitis in
sarcoid usually occurs in the context of chronic systemic (usually pulmonary) sarcoid, and is more
common in Afro-Caribbeans.
Toxoplasma has predominantly posterior segment findings.
Which one of the following concerning Behcet's disease is FALSE?
systemic vasculitis may be successfully treated with colchicine
anterior uveitis is more common in Behcet's disease than posterior uveitis
it is more common among Japanese than Caucasians
a mobile hypopyon is characteristic of Behcet's anterior uveitis
aphthous ulcers develop on mucous membranes including the mouth
Your answer was CORRECT
Explanation
Behcet's disease is much more common among Japanese and individuals from eastern
Mediterranean countries. Posterior uveitis, which can include retinal vasculitis, retinal
hemorrhages, and retinal necrosis, is more common than anterior uveitis in Behcet's disease.
Which is FALSE regarding uveitis associated with juvenile rheumatoid arthritis?
most affected children are rheumatoid factor (RF)-positive
most affected children are girls
most affected children have pauci-articular arthritis
most affected children are antinuclear antibody (ANA)-positive
Your answer was CORRECT
Explanation
Rheumatoid factor is typically absent in patients with uveitis secondary to juvenile rheumatoid
arthritis (JRA/JIA). Also, patients with polyarticular JRA/JIA are less likely to develop uveitis
than those with pauciarticular (fewer than five joints) or monoarticular disease.
Risk factors for ocular involvement in JIA include:
ANA positive
rheumatoid factor negative
pauciarticular type (90% of cases)
female
involvement of lower extremity joints
lack of involvement of the wrists
early age of disease onset
Which of the following medications act(s) by preventing proteolytic release of active HIV
particles?
1. zalcitabine
2. zidovudine
3. didanosine
4. ritonavir
1 and 3
2 and 4
1, 2, 3, and 4
1, 2, and 3
4 only
Your answer was INCORRECT
Explanation
The first three options are reverse transcriptase inhibitors. Ritnavir is a protease inhibitor.
Diagnostic criteria for Behcet's disease include all EXCEPT:
pyoderma gangrenosum
positive Pathergy test
HLA-DR4 antigen
recurrent oral and/or anogenital ulcers
Your answer was CORRECT
Explanation
Figure: Aphthous mouth ulcer in patient with Behcet's disease.
Behcet's is closely associated with HLA-B51.
According to the International Study Group guidelines, for a patient to be diagnosed with Behcet's
disease, the patient must have oral aphthous ulcers (any shape, size, or number at least 3 times in
any 12 months period) along with 2 out of the following 4 hallmark symptoms:
genital ulcers: including anal ulcers and spots in the genital region and swollen testicles or
epididymitis in men
skin lesions: papulo-pustules, folliculitis, erythema nodosum, pyoderma gangrenosum, acne in
post-adolescents not on corticosteroids
eye inflammation: iritis, uveitis, retinal vasculitis, cells in the vitreous
pathergy reaction: papule >2 mm diameter 24-48 hrs or more after needle-prick. The pathergy
test has a specificity of 95% to 100%, but the results are often negative in American and
European patients.
A patient with corneal epithelial deposits, hepatosplenomegaly and bone pain is MOST likely to
have:
Gaucher disease
Lowe syndrome
Cystinosis
Alport syndrome
Alagille syndrome
Your answer was INCORRECT
Explanation
Gaucher disease causes:
accumulation of glucocerebroside
hepatosplenomegaly
bone pain
prominent pingueculae
white deposits in the corneal epithelium
vitreous opacities
para-macular grey ring
A 32-year-old Afro-Caribbean lady presents with unilateral floaters and blurred vision. Slit-lamp
examination reveals anterior chamber flare and cells, iris nodules and vitreous cells. A chest X-ray
reveals hilar lymphadenopathy. She has an enlarged lacrimal gland.
All of the following additional findings are consistent with the likely diagnosis EXCEPT:
granulomatous keratic precipitates
follicular conjunctivitis
scleritis
papillitis
Your answer was CORRECT
Explanation
The presentation is suggestive of sarcoid.
Ocular features may include:
conjunctival nodules
lacrimal gland enlargement
anterior uveitis with Koeppe and Busacca nodules
vitritis
chorioretinal nodules
papillitis
scleritis
keratoconjunctivitis sicca
band keratopathy
secondary glaucoma
Follicular conjunctivitis is commonly found in viral conjunctival infections.
Retinal metastases are most commonly from a primary:
cutaneous melanoma
lung tumor
gastrointestinal tumor
breast tumor
prostate tumor
Your answer was CORRECT
Explanation
Retinal metastases are rare. Cutaneous melanoma is the most common primary tumor.
Reference: Shields et al. Retinal metastasis from systemic cancer. JAMA Ophthalmol. 2014
Nov;132(11):1303-8.
A positive Venereal Disease Research Laboratories (VDRL) test can be caused by all of the
following EXCEPT:
systemic lupus erythematosus
rheumatoid arthritis
antiphospholipid syndrome
Wegener's granulomatosis
syphilis
Your answer was CORRECT
Explanation
VDRL is a test for syphilis which detects antilipoidal antibodies produced by the host during
treponemal infection. However, false positive results can occur. In particular, autoimmune
diseases, such as rheumatoid arthritis and lupus erythematosus can produce similar anti-lipid
antibodies that may give a false-positive reading. Note: Wegener's granulomatosis is associated
with the antineutrophil cytoplasmic antibody not anti-lipoidal antibodies.
Which is the intermediate host for schistosomiasis?
Simulium (black fly)
Chrysops (horse fly)
Ixodes tick
Freshwater snail
Sheep
Your answer was CORRECT
Explanation
The freshwater snail is the intermediate host/vector for schistosomiasis.
Some common vectors:
Vector Disease
Simulium (black fly) onchocerciasis
Chrysops (horse fly) loa loa
Freshwater snail schistosomiasis
Ixodes Lyme disease
Which vessels are typically affected in Takayasu's disease:
small and medium sized arteries
any large vessel with an elastic lamina
renal arterioles
vessels arising from the aortic arch
Your answer was CORRECT
Explanation
Takayasu's disease affects major vessels arising from the arch of the aorta.
A patient visiting from West Africa presents with bilateral photophobia and reduced vision with
floaters. On examination, vision is 6/18 bilaterally. There are live, free-floating filariae visible in
the anterior chamber and evidence of bilateral chorioretinitis.
Which treatment is most likely to be effective?
azithromycin
sulfadiazine
albendazole
ivermectin
Your answer was CORRECT
Explanation
This patient has onchocerciasis. The treatment is ivermectin, which is a broad-spectrum anti-
parasitic agent, traditionally used against worm infestations such as onchocerciasis and filariasis.
Albendazole is effective against cysticercosis - though its use in ocular cysticercosis is not
indicated because dead parasites may induce intense inflammation. Sulfadiazine and azithromycin
are both antibiotics - they are both used in the treatment of toxoplasmosis.
Toxacara canis:
is a trematode
is a nematode
is usually sensitive to flucloxacillin
secretes its eggs in urine
Your answer was CORRECT
Explanation
Toxocara canis is a nematode (roundworm), which is one class of parasitic worm or helminth. The
adult forms are found in the intestines of dogs and cats where they lay eggs that are secreted in the
faeces. Treatment is with oral thiabendazole.
Which of the following is FALSE about louse infections of the eye?
eradication of organisms depends on suffocation, either by bland ointments or paralytic
medications such as eserine
Pediculus capitis is the most common causative organism
the same organisms that infect eyelashes infect pubic hairs
sexual contact is the significant mode of transmission
ocular irritation is due to injection of toxic louse saliva into lid tissue
Your answer was CORRECT
Explanation
Figure: eyelash lice
Phthirus pubis (crab louse) are by far the most common cause of lice infestation of the eye lashes.
Crab louse typically infect the pubic hair and the most common mode of transmission is by sexual
contact. Pediculus capitis (head louse) and pediculus corporis (body louse) may also on rare
occasions infect the eye lashes. See reference: Kumar et al. Eye (2003) 17, 538–539.
All of the following systemic conditions may be associated with necrotising scleritis EXCEPT:
Crohn's disease
polymyalgia rheumatica
Wegener's granulomatosis
polyarteritis nodosa
Your answer was CORRECT
Explanation
PMR is not associated with scleritis.
Systemic associations of scleritis include:
rheumatoid arthritis
polyarteritis nodosa
relapsing polychondritis
Wegener's
inflammatory bowel disease
Reiter's syndrome
Behcet's disease
ankylosing spondylitis (rare)
Rachel, a 24-year-old woman gave birth to an infant who had central nervous system, cardiac and
facial defects. On further questioning, she said that she had taken retinoic acid for acne during her
pregnancy.
In humans, retinoic acid regulates which genes during normal development:
FMR-1 gene
Hox gene
Rb gene
Bcl-2 gene
PAX-3 gene
Your answer was CORRECT
Explanation
This is a case of retinoic acid embryopathy. Retinoic acid is an agent that alters Hox gene
expression.
The homeobox (HOX) or homeotic genes are clusters of mammalian genes involved in the
embryonic development of brain, skeleton and other tissues. Administration of retinoic acid during
pregnancy produces malformations such as cleft lip, cleft palate, and atrial septal defect. Retinoic
acid and its metabolite 9-cis retinoic acid interact with nuclear receptors related to the steroid and
thyroid hormone receptors. This family of proteins function as ligand-dependent transcription
factors. The receptors have separate DNA-binding domains and hormone-binding domains. There
are three retinoic acid receptors (RAR-alpha, beta and gamma) and three receptors for 9-cis
retinoic acid (RXR-alpha, beta and gamma). These receptors recognise response elements in their
target DNA sequences.
The FMR-1 gene is associated with Fragile-X Syndrome. Fragile X syndrome is the most frequent
form of inherited mental retardation - macro-orchidism is a feature of this syndrome. It is a
trinucleotide repeat disorder. In patients with Fragile X syndrome, there is an increase in the size
of an FMR-1 exon containing a CGG repeat, as well as abnormal methylation of a CpG island 250
bp proximal to this repeat.
PAX-3 gene (paired box) gene mutations are associated with Klein-Waardenburg syndrome. This
syndrome is characterised by pigmentary abnormalities such as hair and iris hypopigmentation,
dystopia canthorum and congenital sensorineural hearing loss.
The Rb gene is a tumour suppressor gene encoding the retinoblastoma protein. This is a nuclear
phosphoprotein involved in cell cycle control. The nonphosphorylated form of Rb (which usually
exists in a G0/G1 cell) binds and inhibits several target proteins including the E2F group of
transcription factors. E2F transcription factors activate genes which promote S-phase.
Phosphorylation of Rb releases these proteins allowing S phase to proceed. In this and in other
ways, the Rb gene product plays a role in cell cycle regulation. Mutations in the Rb gene result in
tumourigenesis -retinoblastomas, osteosarcomas and also carcinomas of the breast, colon and
lung.
Bcl-2 is the prototype anti-apoptotic gene. It is a member of a large family of genes - some pro-
apoptotic and some anti-apoptotic in their action. Bcl-2 is bound by a transmembrane anchor to
the outer mitochondrial, nuclear and ER membrane. It prevents the release of cytochrome-c: an
important mediator of the apoptotic pathway. It is a proto-oncogene, i.e. over expression results in
uncontrolled proliferation and neoplasia.
Which statement is FALSE regarding screening for uveitis in JIA:
it is not required for systemic onset JIA
it is required more frequently if ANA positive
it is required more frequently in pauci-articular than poly-articular onset
screening where indicated occurs for at least 5 years or until 12 years of age
Your answer was CORRECT
Explanation
Guidelines for ocular screening in JIA are as follows:
systemic onset: no screening
poly-articular onset: every 9 months
poly-articular onset with ANA positive: every 6 months
pauci-articular onset: every 3 months
pauci-articular onset with ANA positive = every 2 months
Note: Screening, where indicated, should be performed for at least 7 years from the onset of
arthritis or until 12 years of age.
All of the following are examples of inherited cancers EXCEPT:
breast cancer
ovarian cancer
cervical cancer
familial adenomatous polyposis (FAP)
multiple endocrine neoplasia type 2 (MEN-2)
Your answer was CORRECT
Explanation
FAP is inherited in an AD pattern. Multiple benign polyps can occur along the length of the small
and large intestine, with a high risk of these polyps becoming malignant.
About 5% of all breast cancers and approximately 1% of ovarian cancers are caused by a
dominant mutation in the inherited susceptibility genes called BRCA1 and 2.
MEN-2 is a rare genetic syndrome with medullary thyroid cancer, parathyroid adenomas and
phaeochromocytomas.
Note: cervical cancer is associated with prior infection with HPV (human papillomavirus).
What is the most common retinal finding in AIDS?
branch vein occlusion
peri-phlebitis
cotton wool spots
retinal necrosis
venous tortuosity
Your answer was CORRECT
Explanation
Cotton wool spots may occur in over 50% of AIDS patients. The cotton wool spots may
spontaneously resolve and recur.
Which statement regarding sarcoidosis is TRUE?
hilar lymphadenopathy is the most common pulmonary finding
blind conjunctival biopsies have a high yield in patients with presumptive sarcoid
raised serum ACE is specific for sarcoidosis
uveitis occurs in 80% of patients with systemic sarcoidosis
histology shows caseating granulomas
Your answer was CORRECT
Explanation
Hilar lymphadenopathy is the most common pulmonary finding in sarcoidosis. Elevated serum
ACE levels may be seen in any diffuse granulomatous disease and are not specific for sarcoid.
Blind conjunctival biopsies have a characteristically low yield in patients who have only the
presumptive diagnosis of sarcoidosis. Ocular involvement occurs in approximately 25% of
patients with systemic sarcoidosis. Uveitis may be seen in up to 60% of patients with ocular
involvement.
A 44-year-old HIV positive man presents with choroiditis and periphlebitis. The fluorescent
treponemal antibody absorption test is positive.
What is the most appropriate antibiotic treatment for this patient?
penicillin
co-trimaxazole
metronidazole
gentamicin
Your answer was CORRECT
Explanation
The patient has syphilitic chorioretinitis, which should be treated with IV penicillin.
A 42-year-old Eastern European man presents with chorioretinitis. A fluorescent treponemal
antibody test is positive and he is admitted for intravenous penicillin.
What common reaction does the managing physician need to be aware of?
Jarisch-Herxheimer reaction
aplastic anaemia
anaphylactic shock
disseminated intravascular coagulation
Your answer was CORRECT
Explanation
The Jarisch-Herxheimer reaction is seen in half of patients with primary syphilis and up to 90% of
patients with secondary syphilis on IV penicillin treatment. It is due to release of TNF-alpha and
interleukins. It occurs about 8 hours after the first injection and usually consists of mild fever,
malaise and headache lasting several hours. Prednisolone given for 24 hours prior to therapy may
ameliorate the reaction. Penicillin should not be withheld because of the reaction.
A patient visiting from Cameroon presents with bilateral ocular pain and reduced vision with
floaters. On examination, vision is 6/18 bilaterally. There are live, floating filariae visible in the
anterior chamber and bilateral chorioretinitis.
What is the most likely diagnosis?
cryptococcus
toxocariasis
cysticercosis
onchocerciasis
Your answer was CORRECT
Explanation
This patient has onchocerciasis which is caused by the filarial parasite Onchocerca volvulus. It is
transmitted by the bite from the blackfly and results in the migration of millions of tiny worms
(microfilareiae) throughout the body. The microfilariae can be visualised in the anterior chamber.
Signs include anterior uveitis and chorioretinitis (usually bilateral). Treatment is with ivermectin.
A Type 2 diabetic on insulin is having acute onset difficulty breathing and feels nauseous. She has
just had an eye photograph and her skin has an orange tinge. She has very warm peripheries and a
bounding pulse with mild neck swelling. BP is 82/42 and heart rate is 170 bpm.
These features are most in keeping with which diagnosis?
diabetic ketoacidosis
hyperglycaemic hyperosmolar non-ketotic acidosis
hypoglycaemia
anaphylaxis
Your answer was CORRECT
Explanation
An allergy to sodium fluorescein dye used in fundus angiography can result in anaphylactic shock.
What is the intermediate host for schistosomiasis?
pig
man
snail
dog
Your answer was CORRECT
Explanation
The intermediate host for schistosomiasis is the snail.
This question appeared in the 2014 FRCOphth Part 2.
Which is NOT a characteristic of uveitis associated with juvenile idiopathic arthritis:
bilateral
granulomatous
not correlated to joint activity
asymptomatic
Your answer was CORRECT
Explanation
JIA-associated uveitis is typically:
bilateral (70%)
asymptomatic
non-granulomatous
flare-ups are not correlated with joint disease activity
A history of seizures is LEAST compatible with:
ataxia-telangiectasia
Sturge-Weber syndrome
neurofibromatosis
tuberous sclerosis
Your answer was CORRECT
Explanation
Seizures are part of the classic triad in tuberous sclerosis (80% of patients have seizures). Patients
with Sturge-Weber syndrome and associated meningeal haemangioma may have seizure disorders.
Patients with neurofibromatosis also have an increased risk of seizures. However, ataxia-
telangiectasia is not typically related with seizures.
Features of ataxia telangiectasisa (Louis-Bar syndrome):
autosomal recessive inheritance
cerebellar dysfunction: ataxia, oculomotor apraxia, nystagmus
telangiectasia of conjunctiva and the skin of nose, ears, elbows and knees
delayed motor development
immunodeficiency: recurrent respiratory infections
premature graying of hair
cafe-au-lait spots
susceptability to cancer: lymphoma and leukaemia
Which investigation is LEAST likely to be beneficial in a patient suspected of having sarcoidosis?
serum angiotensin-converting enzyme
creatinine kinase
C-reactive protein
serum calcium
Your answer was CORRECT
Explanation
Creatinine kinase is increased in myocyte death, for example in myocardial infarction or myositis.
Patients with xeroderma pigmentosum have a high risk of developing skin cancer. What kind of
genetic deficiency do those patients have?
mismatch repair
protein kinase
ligase
base excision repair
nucleotide excision repair
Your answer was CORRECT
Explanation
Xeroderma pigmentosum is an inherited skin disorder characterised by photosensitivity with
severe sunburn in infancy, the development of numerous pigmented spots resembling freckles,
larger atrophic lesions associated with telangiectasis, and multiple solar keratoses. Transmitted in
an autosomal recessive manner, xeroderma pigmentosa involves a defect in nucleotide excision
repair (NER), leading to deficient repair of DNA damaged by UV radiation and chromosome
breakage. Individuals with this disease develop multiple malignant cutaneous neoplasms at an
early age, and may suffer from severe ophthalmic and neurological abnormalities.
Which of the following skin conditions is classic for Lyme disease:
erythema multiforme
erythema gyratum
condyloma latum
erythema migrans
erythema nodosum
Your answer was CORRECT
Explanation
The first stage of Lyme disease is characterised by a bull's eye skin rash called erythema migrans
which is pathognomic for Lyme disease. It begins 3-30 days following a bite from the Ixodes tick.
Which of the following does NOT apply in hypovolaemic shock:
there is a fall in renin concentration
the pulse rate increases
the blood flow to the brain and kidneys are initially maintained
there is a loss of more than 20% of the blood volume
the urine output decreases
Your answer was CORRECT
Explanation
Renin secretion is increased during hypovolaemic shock to maintain the plasma volume by
conserving water. All other options are true.
Which of the following is NOT a risk factor for the development of uveitis in juvenile idiopathic
arthritis:
ANA positive
RF positive
early onset of JIA
pauci-articular involvement
Your answer was CORRECT
Explanation
Risk factors for ocular involvement in JIA include:
ANA positive
rheumatoid factor negative
pauciarticular type (90% of cases)
female
early onset JIA
HLA-DR 5
involvement of lower extremity joints
lack of involvement of the wrists
Which statement regarding amyloid is FALSE:
it shows an apple green birefringence in polarised light
it appears as extracellular eosinophilic hyaline material
it is a complication of chronic infection
it is made up of calcified protein
it occurs in patients with multiple myeloma
Your answer was CORRECT
Explanation
Amyloid is a fibrillar glycoprotein (not a calcified protein) which appears as extracellular
eosinophilic hyaline material. It shows an apple green birefringence in polarised light.
Amyloidosis may occur in multiple myeloma, in which case the amyloid is made up of the light
chain of immunoglobulin. Secondary amyloidosis occurs in chronic infection.
Which statement about Down's syndrome is FALSE:
it is most commonly caused by chromosomal non-disjunction during meiosis
it is associated with congenital glaucoma
the risk of Down's is associated with the age of the mother
amniocentesis is a more accurate prenatal test for Down's than hormonal tests
Your answer was CORRECT
Explanation
Down's syndrome is associated with cataract, myopia and keratoconus but not glaucoma.
Pyoderma gangrenosum is a feature of:
sarcoidosis
Behcet's disease
Wegener's granulomatosis
polyarteritis nodosa
Your answer was CORRECT
Explanation
Pyoderma gangrenosum can be caused by:
inflammatory bowel including ulcerative colitis and Crohns
rheumatoid arthritis
seronegative arthritis
haematologic disease including leukaemia and myeloma
Wegener's granulomatosis
A young pregnant lady would like to know the risk of having a child with genetic abnormalities.
Her father is affected by haemophilia, which is inherited as an X-linked recessive disease. She is
herself completely asymptomatic. She knows from her last ultrasound test that her baby is a boy.
What is the chance her son is affected?
1 in 4
1 in 6
1 in 16
1 in 2
1 in 8
Your answer was CORRECT
Explanation
Haemophilia is X-linked. The daughter in the question above must inherit her X-chromosome
from her affected father; so she is a carrier. Her male children have a 50% chance of inheriting the
affected X chromosome.
A 40-year-old man comes to see you because he is worried about his family history of
Huntington's disease. His 45-year-old sister is known to be affected and has been given a
molecular diagnosis, but neither parent is affected and both have had a normal gene test.
What is the most likely reason for this inheritance pattern?
non-paternity
non-penetrance in the parents
a new mutation in the sister
anticipation
females are more often affected
Your answer was CORRECT
Explanation
Huntington's disease is an autosomal-dominant neurodegenerative condition caused by a
trinucleotide repeat expansion, and both sexes are equally affected. Less than 1% of cases are due
to new mutations and they are virtually all due to the same abnormal expansion. It is highly
penetrant and onset is typically in middle life. The expansion is more unstable if transmitted
through the paternal line, leading to a younger age of onset in the subsequent generation
(anticipation). However, as the father does not carry the expansion this does not explain the
situation in this family. Non-paternity would therefore have to be seriously considered. Of course,
the situation would have to be handled very delicately. Non-paternity rates are notoriously
difficult to measure. Many studies have found very variable results, from 1-30% or even higher in
some age groups.
The most common sign of Graves' ophthalmopathy is:
superficial punctate keratitis (SPK)
conjunctival injection over the horizontal rectus muscles
diplopia in upgaze
proptosis
lid retraction
Your answer was CORRECT
Explanation
Lid retraction and lid lag are the most common signs in Graves disease. Conjunctival injection and
superficial punctate keratitis are also very common, as is proptosis.
A previously well 54-year-old man develops blurred vision and headache overnight. On
examination, he has normal movements of the right eye, but complete ophthalmoparesis of the left
eye. The left pupil is dilated. There is some sensory loss to light touch and pin prick over the left
cheek. The rest of the neurological examination is normal. Pulse is 80/min, blood pressure 130/87.
Which of these is the LEAST likely anatomical site to explain his problem:
left cavernous sinus
left carotid artery
left posterior communicating artery
brainstem and pons
left superior orbital fissure
Your answer was INCORRECT
Explanation
Loss of all eye movements unilaterally is unusual. It is unlikely to be due to neuromuscular causes
such as myasthenia gravis or Guillain-Barre Syndrome, as both eyes would be affected.
Involvement of the 3rd, 4th and 6th cranial nerves could produce this picture. All these nerves
arise in the brainstem/pons and pass through the cavernous sinus, where they may be affected by
aneurysm or thrombosis. Infection at this site would be another cause. All these nerves pass
through the superior orbital fissure. An aneurysm of the posterior communicating artery causes a
3rd nerve palsy.
Which agent is the drug of choice for prophylaxis of Pneumocystis carinii pneumonia (PCP) in
human immunodeficiency virus (HIV)-positive patients?
oral pentamidine
oral trimethoprim-sulfamethoxazole
oral dapsone
oral pyrimethamine
aerosolized pentamidine
Your answer was CORRECT
Explanation
Bactrim or septrin (trimethoprim-sulfamethozazole) is effective and has a relatively low cost.
Unlike aerosolized pentamidine, it is a systemic treatment and may also be effective for treating
extrapulmonary pneumocystis (e.g., choroidal).
The typical corneal finding in patients with Fabry's disease is:
vortex keratopathy
corneal guttae
anterior membrane dystrophy
central corneal clouding with a clear periphery
superficial peripheral corneal neovascularization
Your answer was CORRECT
Explanation
Figure: Vortex keratopathy
Fabry disease is characterised by:
X-linked recessive inheritance
Deficiency in alpha-galactosidase enzyme
Peripheral neuropathy
GI symptoms
Cardiac and renal impairment
Skin: widespread angiokeratomas
Ocular:
o vortex keratopathy
o cataract
o retinal vessel tortuosity
Other causes of vortex keratopathy:
Fabry's disease
Drugs
o amiodarone
o chlorpromazine
o chloroquine
o indomethacin
o meperidine
o tamoxifen
Which of the following treatments is indicated for Pneumocystic carinii?
Albendazole
dapsone
benzylpenicillin
Amphotericin B
Your answer was CORRECT
Explanation
PCP occurs with CD4 counts less than 200. The differential includes cytomegalovirus,
Cryptococcus, fungi, or other protozoa such as Microsporidia. Treatment includes
trimethoprim/sulfamethoxazole (Bactrim), atovaquone, pentamidine, or dapsone.
Which statement is FALSE regarding sarcoidosis:
elevated serum-ACE levels occur in approximately two-thirds of patients
histologically it is characterized by non-caseating granulomas
hilar and mediastinal adenopathy are common chest x-ray findings
seventh nerve palsy is typically caused by a primary granulomaous neuritis
Your answer was CORRECT
Explanation
Parotid gland infiltration by sarcoid compresses the facial nerve as an innocent bystander (it is not
affected by a primary neuritis). Other options supplied are true.
Fabry's disease is caused by a deficiency of:
alpha galactosidase
beta galactosidase
alpha lecithin cholesterol acyltransferase
alpha L iduronidase
beta glucuronidase
Your answer was CORRECT
Explanation
Fabry's disease is an X-linked recessive condition caused by alpha galactosidase deficiency.
It is characterized by:
conjunctival and retinal vessel tortuosity
cataract: either an anterior capsular/subcapsular or a radial posterior subcapsular cataract
vortex keratopathy
ocular motility abnormalities
The triad of adenoma sebaceum, mental retardation, and seizures is considered pathognomonic
for:
angiomatosis retinae
ataxia-telangiectasia
Sturge-Weber syndrome
tuberous sclerosis
neurofibromatosis
Your answer was CORRECT
Explanation
Figure. Adenoma sebaceum associated with tuberous sclerosis.
The triad presented in the question above is known as Vogt's triad and is present in 30% of
patients with tuberous sclerosis.
Tuberous sclerosis is characterised by:
cerebral cortical tubers, after which the disease is named
astrocytic hamartomas of retina and brain (often calcified)
seizures
mental retardation
renal angiomyolipomas
adenoma sebaceum
cafe-au-lait spots
ash leaf depigmentation
shagreen patches
periungual fibromas
A 30-year-old Japanese man presents with blurred vision. On examination, he has a hypopyon,
mild AC cells, and evidence of peri-phlebitis. He complains of arthritis, acne on his face and back
and has red plaques on his shins.
Which statement concerning the therapeutic treatment of this condition is TRUE?
periocular steroids alone are usually effective in preventing a relapse of ocular inflammation
anti-TNF agents are useful in the acute stages and should be tapered rapidly
colchicine may be helpful in preventing recurrences
cyclosporine may be useful in this condition, but liver toxicity is a common limitation
oral prednisolone is usually effective in preventing a relapse of ocular inflammation
Your answer was CORRECT
Explanation
This patient has Behcet's disease. Commonly used drugs in this condition include anti-TNF agents
(e.g. etanercept or infliximab), systemic corticosteroids, interferon-alpha, cyclosporine,
azathioprine and colchicine.
While systemic corticosteroids may be initially effective in treating the ocular inflammation in
Behcet's they do not prevent long-term progression.
For ocular disease, anti-TNF agents are gaining widespread use. Azathioprine has also been
widely accepted as an initial treatment choice. For severe eye disease (significant drop in visual
acuity, retinal vasculitis, or macular involvement), either cyclosporine A or infliximab (anti-TNF)
may be used in combination with azathioprine and corticosteroids.
Cyclosporine is mainly associated with renal (not liver) toxicity.
Colchicine, although becoming a more dated choice compared to biologics, has been shown to be
useful in reducing recurrences.
Which of the following is responsible for acute extraocular muscle oedema in thyroid orbitopathy?
hypertrophy of blood vessels
glycosaminoglycans
keratin deposition
hypertrophy of myocytes
Your answer was CORRECT
Explanation
GAG are secreted by inflammatory cells in the extraocular muscles in thyroid orbitopathy. This
causes an osmotic gradient in the muscle, and oedema ensues.
There is a well documented association between superior limbic keratoconjunctivitis and which
systemic condition:
thyroid disease
inflammatory bowel disease
systemic lupus erythematosus
rheumatoid arthritis
Your answer was CORRECT
Explanation
Figure: Superior limbic keratoconjunctivitis
About 50% of patients with superior limbic keratoconjunctivitis (SLK) have some form of thyroid
disease. Treatment of the thyroid disorder, however, has little effect on the SLK.
Which is TRUE of Toxoplasma gondii infection:
it can be transmitted through dog faeces
it is more common in immunocompromised patients than cytomegalovirus
acquired infection usually causes a severe acute illness
it can be transmitted from undercooked meat
Your answer was CORRECT
Explanation
Toxoplasma gondii is the commonest protozoal parasite to affect the eye. It is transmitted from
ingestion of undercooked meat or the ingestion of soil contaminated with infected cat faeces.
(Toxocara is transmitted through dog faeces). Congenital infection may result in severe symptoms
including hydrocephalus, intracranial calcification and mental retardation. Acquired infection is
often asymptomatic. It is less commonly associated with immunocompromise than CMV retinitis.
Which systemic feature may be found in a patient who presents with the condition shown above:
decreased thyroid stimulating hormone
increased urinary cortisol
hyperextensible joints
vesicular skin rash
Your answer was CORRECT
Explanation
The figure above shows superior limbic keratoconjunctivitis. Thyroid abnormalities have been
associated with SLK.
Superior limbal keratoconjunctivitis is characterised by:
superior limbal hyperemia
mild keratitis
superior palpebral conjunctival papillary reaction
micropannus
filaments
rose bengal or lissamine green are useful dyes to show affected conjunctiva
may be associated with thyroid dysfunction
Treatment: 0.5% to 1.0% silver nitrate solution, pressure patching, mechanical scraping,
conjunctival resection, or bandage contact lens
What is the most common antibody in systemic lupus erythematosus?
anti-Ro
c-ANCA
anti-dsDNA
p-ANCA
Your answer was CORRECT
Explanation
Anti-nuclear antibody (ANA) testing forms the mainstay of serological testing for SLE. Sub-types
of ANA include anti-dsDNA, anti-histone, anti-RNP and anti-Ro. Of these, anti-dsDNA is the
most common in SLE and the most specific, being present in 70% of SLE cases, but only 0.5% of
the general population.
This question came in the 2014 FRCOphth.
Which condition can be inherited as X-linked dominant?
Alport syndrome
Lowe syndrome
Schubert-Bornschein syndrome
blue-cone monochromatism
juvenile retinoschisis
Your answer was CORRECT
Explanation
Alport's syndrome can be inherited in both X-linked recessive (80%) and X-linked dominant (5%)
forms. Where X-linked recessive, heterozygous females may still experience haematuria - a
separate condition termed 'thin basement membrane nephropathy.' Reference: NIH National
Library of Medicine.
All other options above are X-linked recessive.
X-linked dominant conditions are rare, with examples being:
Alport syndrome (5% of cases)
Aicardi syndrome
incontinentia pigmentii
Mikulicz's syndrome refers to the combination of chronic dacryoadenitis with:
enlargement and inflammation of the parotid glands
rheumatoid arthritis
keratoconjunctivitis sicca
systemic lupus erythematosus
dacryocele
Your answer was CORRECT
Explanation
Chronic dacryoadenitis is sometimes accompanied by inflammation and swelling of the parotid
and salivary glands, which is referred to as Mikulicz's syndrome. This may be a primary, benign
enlargement or secondary to systemic disease, such as tuberculosis, sarcoidosis, lymphoma, and
Sjogren's syndrome.
The systemic amyloid associated with rheumatoid arthritis is derived from:
immunoglobulin light chains
serum AA protein
polypeptide hormones
prealbumin
Your answer was CORRECT
Explanation
Amyloid deposition may be derived from a number of sources depending on the pathology. It may
occur secondary to systemic disease such as rheumatoid, in which case it is derived from serum
AA protein, an acute phase reactant. It may occur from light chain immunoglobulins as in
myeloma and macroglobulinaemia. It may occur from polypeptoid hormones as in carcinoma of
the thyroid. In Alzheimers it is derived from prealbumin.
Most cases of Lyme disease present in:
sping
winter
summer
autumn
Your answer was CORRECT
Explanation
Nearly 75% of Lyme disease cases occur in the summer months.This is related to the life-cycle of
the Ixodes tick vector.
A lady consults the geneticist. Her brother is suffering from haemophilia. She wants to know what
is the chance that her daughter will be a carrier of haemophilia assuming that her husband is
normal?
1 in 1
1 in 2
1 in 16
1 in 8
1 in 4
Your answer was CORRECT
Explanation
Haemophilia is an X-linked condition. Since this lady's brother is affected, there is a 50% chance
that she has also inherited the affected chromosome. In turn, there is a 50% chance that she would
pass this on to her daughter, so the probability is:
0.50 x 0.50 = 0.25
Hutchison's triad refers to each of the following signs EXCEPT:
sensorineural deafness
saddle nose
notched teeth
interstitial keratitis
Your answer was CORRECT
Explanation
Figure: Notched teeth in congenital syphilis, which is part of Hutchison's triad.
Saddle nose occurs in some patients with syphilis but this feature is not part of Hutchison's triad.
Hutchison's triad consists of:
interstitial keratitis
notched teeth (shown in image above)
sensorineural deafness
A 40-year-old Malaysian man presents with apthous mouth ulcers, arthritis, bilateral mobile
hypopyon, periphlebitis and occlusive arteritis.
All of the following are associated with this condition EXCEPT:
diarrhea
acne-form rash
cardiac conduction defects
back pain
Your answer was INCORRECT
Explanation
Behcet's disease may be associated with painful, hypopyon iridocyclitis. The eye can also be
remarkably quiet in the presence of a hypopyon in Behcet's disease, unlike in HLA-B27-
associated disease, in which pain and photophobia dominate as symptoms. A strong association
with HLA-B5/B51 is often present in Behcet's disease.
Systemic features of Behcet's include:
acne
erythema nodosum
aphthous ulcers
genital ulcers
interstitial lung changes
pulmonary artery aneurysm (pathognomonic finding)
Treatment is with systemic and periocular corticosteroids. Colchicine may be useful in reducing
recurrences of the disease. Cyclosporine and cytotoxic agents may also be required
Which one of the following inherited diseases is due to mutation in mitochondrial DNA?
Noonan's syndrome
Leber's optic neuropathy
Alport's syndrome
Fabry's disease
Marfan's syndrome
Your answer was CORRECT
Explanation
Prevalence of mitochondrial diseases equals 1:10000 of live-born infants. Mutations of
mitochondrial DNA (mtDNA) are their most frequent cause. The diseases often result in
encephalomyelopathy, cardiomyopathy, vision disorders, dysacusis and metabolic disorders.
Despite numerous studies problems associated with mitochondrial diseases have not been
completely solved yet.
Examples of mitochondria inheritance include:
LHON (Leber's Hereditary Optic Neuropathy) was the first described disease associated with
hereditary point mutations in mtDNA. The disease is characterised by subacute loss of
binocular vision with a lesion of the central field of vision, improper colour vision and atrophy
of optic nerve.
Kearns-Sayre syndrome which is caused both by deletions and duplications of mtDNA. The
onset of disease symptoms is observed before 20 year of age. Short stature, pigmentary retinal
degeneration, ophthalmoplegia, ptosis, ataxias, disturbances in conduction in heart muscle,
diabetes, and hearing loss occur in the syndrome.
CPEO (Chronic Progressive External Ophthalmoplegia) which may occasionally occur as a
result of de novo mutation, may be maternally inherited (mt tRNA mutations) or can be of
autosomally dominant inheritance. Symptoms of CPEO are ptosis, myopathy, depression,
cataract and ketoacidosis.
Note: Alport's syndrome, Noonan's syndrome, Fabry's disease and Marfan's syndrome are diseases
caused by mutations in nuclear and not mitochondrial DNA.
The form of the Toxoplasma gondii organism that is responsible for stimulating inflammation
is/are:
cyst
tachyzoite
bradyzoite
pseudo-hyphae
Your answer was CORRECT
Explanation
Toxoplasma organisms tend to be dormant in the cyst form in the intermediate human host. In the
encysted form, several dormant organisms may be found. These relatively inactive organisms
within the cysts are called bradyzoites. Once bradyzoites are released and become free, they are
called tachyzoites. These metabolically energized protozoans are responsible for stimulation of
intraocular inflammation.
All of the following are unfavorable prognostic factors for graft survival after penetrating
keratoplasty EXCEPT?
dry eye
corneal vascularisation
small graft size
young age
Your answer was CORRECT
Explanation
Larger grafts have an unfavourable prognosis compared to smaller grafts because of a greater risk
of rejection (closer proximity to the limbus). Other options supplied are true.
Which systemic condition is associated with Fuchs' heterochromic cyclitis:
Parry-Romberg syndrome
Raynaud's syndrome
Hashimoto's thyroiditis
Parinaud oculoglandular syndrome
Your answer was CORRECT
Explanation
Parry-Romberg syndrome (hemifacial atrophy) is found in a small proportion of cases of Fuch's
heterochromic cyclitis.
Which is FALSE regarding Giant cell arteritis:
biopsy reveals patchy vessel involvement
Giant cells are found mostly in the internal elastic lamina
there is often obliteration of the artery lumen
it can affect the central retinal arteries and retinal arterioles
Your answer was CORRECT
Explanation
Giant cell arteritis can affect the cerebral arteries, the ophthalmic artery and the central retinal
artery but it will not affect retinal arterioles because they do not possess an elastic layer.
Which is FALSE regarding Giant cell arteritis:
biopsy reveals patchy vessel involvement
Giant cells are found mostly in the internal elastic lamina
there is often obliteration of the artery lumen
it can affect the central retinal arteries and retinal arterioles
Your answer was CORRECT
Explanation
Giant cell arteritis can affect the cerebral arteries, the ophthalmic artery and the central retinal
artery but it will not affect retinal arterioles because they do not possess an elastic layer.
Which one of the following concerning ocular tuberculosis is TRUE?
skin testing is valuable in the diagnosis of tuberculous uveitis but may need to be repeated
with higher concentrations of purified protein derivative (PPD)
treatment of choice for tuberculous uveitis is topical and/or systemic corticosteroid
eye disease cannot be encountered in the setting of a normal chest x-ray
the ocular inflammation reflects a hypersensitivity reaction without active infection
Your answer was CORRECT
Explanation
Tuberculous bacilli may be found histopathologically in eyes with tuberculous uveitis.
Tuberculous uveitis may be present even with a normal purified protein derivative (PPD) test (i.e.
Mantoux test) and a normal chest x-ray. For these cases, a second strength skin test (250
tuberculin units per 0.1ml intradermal injection instead of the usual 5 tuberculin units per 0.1ml)
may be positive. Note: systemic corticosteroids may cause a dangerous flare-up in otherwise
quiescent tuberculosis.
Part 2 Premium – Tutor Mode
Question 8 of 10
Score: 80 %
A 63-year-old Type 2 diabetic patient develops cystoid macula oedema after cataract surgery.
Which of the following medications may be contributing to this?
metformin
simvastatin
amlodipine
rosiglitazone
Your answer was INCORRECT
Explanation
Several published reports have associated the use of rosiglitazone with the development or
exacerbation of macular oedema.
This question came in the FRCS (Glasgow) October 2014 exam.
Which of the following statements about toxocariasis is FALSE?
the uveitis is typically well controlled once the organism has been treated with anti-parastic
agents
there may be an associated peripheral eosinophilia
the infectious cycle in humans generally starts with the consumption of faecally contaminated soil
the condition may present as a peripheral granuloma in an otherwise quiet eye
Your answer was CORRECT
Explanation
The infectious cycle generally starts with a child consuming sand or dirt contaminated by the
faeces of a house pet that ingested the organism (dogs more often than cats). A peripheral
granuloma with overlying vitreous opacification and chronic uveitis are typical findings in
toxocariasis. The uveitis may die down and leave a quiescent peripheral granuloma. The
inflammation in the eye often flares after the larva dies, and corticosteroids are the drug of choice
in the treatment of this condition.
The most common organism involved in mycotic ocular infections in the southern half of the
United States is:
Alternaria
Fusarium
Aspergillus
Penicillium
Candida
Your answer was CORRECT
Explanation
Therapy for ocular mycosis is heavily dependent on geographic location. In the southern (tropical)
United States Fusarium spp predominate, while in the northern United States Candida spp is the
most common.
Which of the following is NOT a trinucleotide repeat disorder?
Fragile X syndrome
Myotonic dystrophy
Huntingdon's disease
Hurler's syndrome
Friedreich's ataxia
Your answer was INCORRECT
Explanation
Trinucleotide repeat disorders include:
Fragile X
Myotonic dystrophy
Huntington chorea
Friedreich ataxia
Spinocerebellar ataxia
Spinobulbar muscular atrophy
Machado-Joseph ataxia
Trinucleotide repeat disorders are characterised by a three base pair sequence that repeats, e.g.
(GCC)n. Trinucleotide repeats can exist in a normal range, which is usually less than 35 repeats,
but if they become unstable they can expand into a mutant range, normally greater than 50 repeats.
There are 2 types, type I and II. Type I involves expansion in a regulatory area, type II does not.
Note that these diseases show anticipation: they become more severe, and occur earlier in
successive generations.
Which of these are well-recognised late complications of trisomy 21 (Down's syndrome)?
glioblastoma
ischaemic heart disease
Alzheimer's dementia
Addison's disease
Your answer was INCORRECT
Explanation
Individuals with Down's syndrome have a greatly increased risk of developing Alzheimer's
dementia. Virtually 100% will show neuropathological changes by the age of 35.
The most common ocular manifestation of cryptococcal infection is:
membranous conjunctivitis
lid abscesses
ulcerative keratitis
endogenous endophthalmitis
orbital cellulitis
Your answer was CORRECT
Explanation
The most common ocular manifestation of cryptococcal infection is endogenous endophthalmitis.
A patient with a previous history of peripheral ulcerative keratitis presents with a right orbital
mass causing proptosis. The patient has a medical history of left elbow arthritis. A chest X-ray
reveals a pulmonary nodule.
What is the most likely diagnosis?
Wegener's granulomatosis
Churg-Strauss
Systemic lupus erythematosus
Sarcoidosis
Your answer was CORRECT
Explanation
Wegener's granulomatosis is by some distance the most common systemic inflammatory cause of
an orbital mass. The clinical features in this case of PUK, a pulmonary nodule and arthritis, are all
in keeping with Wegener's. Systemic lupus can cause many of these features, but orbital
involvement is much rarer by comparison to Wegener's. Sarcoidosis is unlikely as it does not
cause PUK, is less likely to affect the orbit and lung involvement is usually in the form of fibrosis
or hilar lymphadenopathy, not a nodule. Churg-Strauss can cause orbital involvement and PUK,
but again, it is a less common cause than Wegener's, and is usually characterized by eosinophilia,
asthma and mainly respiratory and renal symptoms.
This question came in the 2014 FRCOphth.
Which one of the following regarding onchocerciasis is FALSE?
microfilariae, released by adult worms, penetrate the eye by both direct invasion and
haematogenous spread
skin findings are rare in this condition
chorioretinal and optic atrophy are common in advanced disease
microfilariae may be seen swimming in the anterior chamber and may induce a severe anterior
uveitis with glaucoma and cataract
onchocerca volvulus is transmitted by the bite of the Simulium black fly
Your answer was CORRECT
Explanation
The larvae of Onchocerca volvulus form subcutaneous nodules when they develop into mature
worms. This is one manifestation of onchodermatitis.
A patient has been diagnosed with Marfan's syndrome. Which gene mutation is responsible for
this condition?
Hexosaminidase
Galactosidase
Cyclooxygenase
Mitochondria
Fibrillin
Your answer was CORRECT
Explanation
Marfan's syndrome is characterised by a triad of features:
long, thin extremities frequently associated with other skeletal changes, such as loose joints
and arachnodactyly
reduced vision as the result of dislocations of the lenses (ectopia lentis)
aortic aneurysms that typically begin at the base of the aorta
Most patients with Marfan's can be identified by detection of mutations in the fibrillin gene.
A patient has frontal bossing, hearing impairment, optic atrophy and angioid streaks.
What is the most likely diagnosis?
Sickle-cell disease
Pseudoxanthoma elasticum
Addison's disease
Acromegaly
Paget's disease
Your answer was CORRECT
Explanation
Paget's disease involves excessive and disorganized resorption and formation of bone. Features
include: bowing of the tibias, obstruction of cranial foramina (leading to deafness, optic atrophy,
ocular motility defects, cranial nerve palsies etc) and bony deformity especially of the skull. It is a
known cause of angioid streaks. Treatment is with bisphosphonates.
Which is FALSE regarding erythema multiforme major?
systemic corticosteroids are useful in acute treatment
the classic mucous membrane lesion resembles a target lesion or bull's-eye
lysis of symblephara may reduce ultimate scarring
can be caused by mycoplasma and adenoviral infection
Your answer was CORRECT
Explanation
In erythema multiforme major (Stevens-Johnson syndrome) mucous membrane lesions are
bullous. Skin lesions are macular and consist of a target lesion: red center, inner pale annulus,
outer red annulus.
Which organism has NOT been implicated in triggering Reiter's syndrome?
Chlamydia
Gonorrhea
Yersinia
Ureaplasma urealyticum
Your answer was CORRECT
Explanation
Reiter's syndrome may occur after dysentery or after non-gonococcal urethritis.
Causes of Reiter's syndrome include:
Post-urethritis
o Ureaplasma urealyticum
o Chlamydia trachomatis
Post-dystentry
o Shigella
o Salmonella
o Yersinia
A Type 1 brittle diabetic on insulin collapses unconscious in the ward from hypoglycaemmia.
How would you treat this patient?
20% dextrose iv
5% dextrose iv
10% dextrose iv
50% dextrose iv
Your answer was CORRECT
Explanation
A patient with hypoglycaemia who is awake and can tolerate fluids can be given 15g of simple
carbohydrate orally (60 calories). If the patient cannot tolerate oral intake, they should be given 1
ampoule of 50% Dextrose iv. If there is no iv access, subcutaneous or intra-muscular glucagon
1mg should be administered.
This question appeared in the 2014 FRCOphth Part 2.
All of the following are true regarding familial cancer syndromes EXCEPT:
they tend to be more common in certain ethnic groups
they can be inherited in an autosomal dominant or recessive manner
they are easily distinguishable from sporadic cancers on histology
they tend to present at a younger age than sporadic cancers
these syndromes make up a small proportion of all cancers
Your answer was CORRECT
Explanation
The following factors suggest a familial basis to cancer:
Earlier age of onset than would be expected for a particular tumour
Several close relatives affected
More than one primary tumour in the same individual
A pattern of cancer fitting into a recognised cancer syndrome
It is usually not possible to distinguish a sporadic cancer from a genetic cancer from the histology.
Most cancer syndromes are inherited as AD but a few are AR, e.g. Ataxia telangectasia.
Infectious scleritis can be caused by all of the following EXCEPT:
Tuberculosis
Leprosy
Syphilis
Chlamydia
Herpes zoster
Your answer was CORRECT
Explanation
Half of patients with scleritis have associated systemic disease.
Scleritis is associated with:
infectious diseases:
o syphilis
o tuberculosis
o herpes zoster
o leprosy
autoimmune diseases
o rheumatoid arthritis (most common association)
o Wegener's granulomatosis
o systemic lupus erythematosus
o relapsing polychondritis
o polyarteritis nodosa
o spondyloarthropathies e.g. ankylosing spondylitis, reactive arthritis, psoriatic arthritis
o inflammatory bowel disease
metabolic diseases
gout
All are true of acanthoemeba EXCEPT:
it is very difficult to culture
it is sensitive to 3rd generation cephalosporins
it is a free-living protozoon
it is more common in soft contact lens wearers
Your answer was CORRECT
Explanation
Acanthoemeba corneal involvement is poorly responsive to conventional antibiotics, including the
latest generation cephalosporins. Treatment requires propamidine isethionate, chrohexidine and
polyhexamthyl biguanide.
All are true of ocular inflammation caused by cysticercosis EXCEPT:
free-floating cysts may be visible in the anterior chamber
medical therapy with albendazole is first-line treatment
subretinal cysts may occur giving rise to retinal detachment
the causative organism is the larval form of the pork tapeworm
Your answer was CORRECT
Explanation
Albendazole is effective against cystercosis but it is NOT indicated for ocular disease since dead
parasite may induce inflammation.
Which of the following medications acts by preventing synthesis of viral DNA from RNA?
1. zalcitabine
2. zidovudine
3. didanosine
4. ritonavir
1, 2, and 3
2 and 4
1, 2, 3, and 4
1 and 3
4 only
Your answer was CORRECT
Explanation
Zalcitabine, zidovudine and didanosine are all reverse transcriptase inhibitors. This group of drugs
acts by preventing synthesis of viral DNA from RNA. They were the first available and front-line
class of agents in the fight against human immunodeficiency virus (HIV). Ritonavir is a protease
inhibitor.
Which of the following tissues may give rise to Kaposi's sarcoma?
1. eyelid skin
2. choroid
3. conjunctiva
1 only
1, 2, 3
2 and 3
1 and 2
1 and 3
Your answer was CORRECT
Explanation
Kaposi's sarcoma may be noted on the eyelid skin or conjunctiva. Skin lesions usually appear as
non-tender, elevated, purple nodules. Conjunctival involvement is manifested by red
subconjunctival masses.
In patients with HIV, cytomegalovirus (CMV) retinitis usually occurs with CD4 counts:
less than 100 cells/mm3
200-500 cells/mm3
less than 500 cells/mm3
less than 50 cells/mm3
Your answer was CORRECT
Explanation
CMV retinitis usually occurs with CD4 counts less than 50 cells/mm3.
All of the following conditions are caused by Chlamydia trachomatis EXCEPT?
Adult inclusion conjunctivitis
Ligneous conjunctivitis
Trachoma
Lymphogranuloma venereum
Your answer was CORRECT
Explanation
Figure: sub-tarsal scarring in trachoma
Chlamydia trachomatis serotypes A to C is considered the leading cause of blindness worldwide.
Features of trachoma include (early to late):
follicular conjunctivitis (early stages)
tarsal scarring and fibrosis
cicatricial changes
trichiasis and entropion
corneal scarring
blindness
Chlamydia trachomatis serotypes D to K is associated with adult inclusion conjunctivitis, which
can cause a chronic follicular conjunctivitis.
Lymphogranuloma venereum is caused by infection with Chlamydia trachomatis serotypes L1,
L2, or L3. This disease does not usually affect the eyes.
Ligneous conjunctivitis can result from any ocular infection that produces a vigorous
inflammatory response with exudation of fibrin and formation of a hard, woody membrane.
Which of the following is LEAST likely to be associated with Crohn's disease?
choroiditis
peripheral corneal infiltrates
conjunctivitis
episcleritis
Your answer was CORRECT
Explanation
Ocular manifestations of Crohn's:
conjunctivitis
episcleritis
scleritis
peripheral corneal infiltrates
retinal periphlebitis
Ocular manifestations of Ulcerative Colitis:
conjunctivitis
episcleritis
scleritis
peripheral corneal infiltrates
papillitis (rare)
multi-focal choroiditis (rare)
retinal vasculitis (rare)
Marfan's syndrome is caused by a defect in:
elastin
Type 1 collagen
fibrillin
Type 2 collagen
Type 4 collagen
Your answer was CORRECT
Explanation
Marfan's: defective fibrillin
Ehlers-Danlos: defective Type 1 (or Type 3) collagen
Stickler's: defective Type 2 (or Type 9) collagen
Alport's: defective Type 4 collagen
A couple attends genetic screening for an inherited condition that has been observed in the
husband's family. More severe cases of so-called anticipation have been observed.
What is the most likely condition?
Myotonic dystrophy
Haemophilia B
Achondroplasia
Polycystic kidney disease
Marfan's syndrome
Your answer was INCORRECT
Explanation
Anticipation is the tendency for a genetic disorder to become more severe and present at an earlier
age in successive generations. It is usually the result of an unstable triplet repeat expansion in a
gene and can show 'parent of origin' effect. Examples of genetic disorders that show anticipation
are fragile-X syndrome, Huntington's disease, myotonic dystrophy (MD) and some of the
spinocerebellar ataxias. All these conditions are caused by the expansion of a triplet repeat
expansion within the gene. This repeat can undergo expansion during meiosis such that the
children of an affected individual tend to inherit a larger expansion that results in more severe
disease.
Myotonic dystrophy is an autosomal dominant disorder caused by the expansion of a cytosine-
thymine-guanine (CTG) repeat that lies at the 3' end of the dystrophia myotonica-protein kinase
(DMPK) gene on 19q13.3. Normal individuals have fewer than 35 CTG repeats. Affected
individuals with MD have more than 50 CTG repeats. A CTG expansion of 50 to 80 repeats is
associated with early-onset cataracts. Affected individuals with classical MD have larger CTG
expansions. The most severe form of MD is congenital MD, which presents at birth with
intrauterine growth retardation, severe hypotonia, muscle weakness and feeding difficulties.
Children with congenital MD have developmental delay and learning difficulties and develop
myotonia in their teens. Affected individuals with congenital MD have very large CTG expansions
with 500 to 2000 repeats. Expansions of this size are almost always maternally transmitted.
Which of the following Chlamydial serotypes is associated with trachoma in humans:
D
A
Your answer was CORRECT
Explanation
Figure: sub-tarsal fibrosis in trachoma
Chlamydia trachomatis serotypes A, B, and C cause trachoma. Serotypes D through K cause
genital chlamydia and adult inclusion conjunctivitis, while serotypes L1 through 3 cause
lymphogranuloma venereum.
The second most frequently involved extraocular muscle in Graves' ophthalmopathy is the:
medial rectus
lateral rectus
superior rectus
inferior oblique
inferior rectus
Your answer was CORRECT
Explanation
In descending order from most to least frequently involved in TED:
inferior rectus
medial rectus
superior rectus
lateral rectus
NOTE: remember the pnemonic I M Stuart Little
Thymic aplasia is a feature of:
Sturge-Weber syndrome
von Hippel-Lindau syndrome
Wyburn-Mason syndrome
Louis-Bar syndrome
Your answer was CORRECT
Explanation
Louis-Bar, or ataxia-telangiectasia is characterised by:
autosomal recessive inheritance
cerebellar dysfunction: ataxia, oculomotor apraxia, nystagmus
telangiectasia of conjunctiva and the skin of nose, ears, elbows and knees
delayed motor development
immunodeficiency including thymic aplasia: recurrent respiratory infections
premature graying of hair
cafe-au-lait spots
susceptability to cancer: lymphoma and leukaemia
A 35-year-old man presents with severe hypertension and is found to have a unilateral
phaeochromocytoma. His father died from metastatic renal-cell carcinoma aged 48 and previously
had surgery for a brain tumour.
What is the most likely underlying genetic condition?
von Hippel-Lindau
Neurofibromatosis type 1
Multiple endocrine neoplasia type 2A
Hereditary haemorrhagic telangiectasia
Neurofibromatosis type 2
Your answer was CORRECT
Explanation
Genetic conditions associated with phaeochromocytomas include:
neurofibromatosis type 1 (NF1)
multiple endocrine neoplasia type 2 (MEN2)
von Hippel-Lindau syndrome (VHL)
Note: all are autosomal-dominant.
Individuals with NF1 can be diagnosed using criteria based on the clinical features of cafe-au-lait
spots, axillary freckling, Lisch nodules and neurofibromas. The most serious complications in
adults with NF1 are hypertension due to either renal artery stenosis or phaeochromocytomas and
malignant transformation of the neurofibromas.
MEN2, due to mutations in the RET gene, is characterised by medullary thyroid cancer,
parathyroid hyperplasia and phaeochromocytomas (type 2A) and rarely with a marfanoid habitus
and mucosal neuromas (type 2B).
Complications of the VHL syndrome include phaeochromocytomas, renal-cell carcinomas and
retinal and cerebellar haemangioblastomas, which can cause intracerebral haemorrhage. VHL
syndrome is therefore the most likely genetic condition in this case and the patient and other close
family members should be screened for haemangioblastomas and renal-cell carcinomas in addition
to phaeochromocytomas. DNA analysis will detect VHL gene mutations in over 90% of VHL
families.
Phaeochromocytomas are not associated with hereditary haemorrhagic telangiectasia (HHT) or
neurofibromatosis type 2 (NF2). However, brain tumours (vestibular schwannomas, gliomas,
meningiomas) are a feature of NF2 and cerebral arteriovenous malformations are sometimes seen
in patients with HHT.
Which of the following is associated with Wegener's granulomatosis:
multinucleate giant cells in the internal elastic lamina
a linear deposition of IgG along the basement membrane
medium sized vasculitis with fibrinoid necrosis
small vessel vasculitis with granulomatous infiltration
Your answer was CORRECT
Explanation
Wegener's is characterized histologically by small vessel vasculitis with necrosis and
granulomatous inflammation.
Which one of the following statements concerning toxoplasmosis is TRUE?
the manifestations are strictly posterior
involvement of retinal vessels is inconsistent with toxoplasmosis
vitritis is a sine qua non of ocular toxoplasmosis
sheep are the definitive hosts of Toxoplasma gondii
ELISA for anti-toxo antibodies is important in the diagnosis of atypical lesions
Your answer was CORRECT
Explanation
Granulomatous inflammation of the anterior segment can occur in toxoplasmosis. Perivasculitis
near active retinal lesions is common (kyrieleis arteriolitis). The classic lesion of toxoplasmosis is
exudative focal retinitis. The definitive host for Toxoplasma gondii is the cat, where it is found as
an intestinal parasite.
A patient with Behcet's disease is most likely to have which immune marker?
HLA DR2
HLA A29
HLA B27
HLA DR4
HLA B51
Your answer was CORRECT
Explanation
Behcet's disease is correlated with HLAB51.
Which of the following concerning ocular toxocariasis are true?
1. The definitive host for the parasite is the dog or cat
2. Ingested ova initially take up residence in the liver and lung
3. Features may include chronic endophthalmitis and localised macular granuloma
4. Inflammation is usually unilateral
1, 2, and 3
1, 2, 3, and 4
1 and 3
4 only
2 and 4
Your answer was CORRECT
Explanation
Toxocara canis is an intestinal parasite of dogs and cats. Dogs are more commonly implicated in
human infections. After ingestion of ova, larvae are spawned that will penetrate the intestinal wall
and take up residence in the liver and lungs. From there, larvae can disseminate to any organ,
including the eye. Eye involvement is usually unilateral.
In a patient with AIDS and cytomegalovirus retinitis intravenous or intravitreal antiviral induction
therapy is given for:
14 days
7 days
5 days
2 days
Your answer was CORRECT
Explanation
Either IV or intravitreal antiviral therapy may be used for the treatment of cytomegalovirus
retinitis. Induction is typically carried out for 14 days. Once the retinitis is under control,
maintenance therapy is given indefinitely.
All are true of presumed ocular histoplasmosis syndrome EXCEPT:
it never occurs during active, systemic histoplasmosis
it is characterised by multiple atrophic spots in the mid-peripheral retina
during the acute stage there is only mild vitritis
choroidal neovascularisation is a recognised complication
Your answer was CORRECT
Explanation
POHS is characterised by absence of intra-ocular inflammation, multiple atrophic spots in the
mid-peripheral retina, and peripapillary atrophy. CNV is a recognised complication. POHS never
occurs during active, systemic histoplasmosis. It is thought to represent an immunologic-mediated
response in individuals previously exposed to the fungus.
All are true of sarcoid EXCEPT:
it may cause papilloedema
it may cause choroiditis and retinitis
it is a caseating, granulomatous inflammatory condition
it may cause intermediate uveitis
Your answer was CORRECT
Explanation
Sarcoid is a non-caseating granulomatous inflammatory condition. Other options are true.
Which of the following are typical histological changes seen in benign hypertension:
hyalinization and intimal proliferation of the muscular media of the medium sized arteries
and arterioles
fibrinoid necrosis of small arteries and arterioles
loss of endothelial cells of arterioles
formation of new vessels
Your answer was CORRECT
Explanation
Intimal proliferation and hyalinization of the muscular media are the two histological features
commonly seen in benign hypertension especially in the medium-sized renal arteries and renal
arterioles. Fibrinoid necrosis of small arteries and arterioles are seen in malignant hypertension.
Loss of endothelial cells of arterioles occur in diabetes mellitus.
A 40-year-old man develops jerky movements affecting various parts of his body. His father died
at the age of 55 and had been diagnosed as having Huntington's disease.
Which of the following genetic abnormalities is responsible for this condition?
termination mutation
splicing mutation
frame-shift mutation
triplet-repeat mutation
point mutation
Your answer was CORRECT
Explanation
Huntington's disease, an autosomal-dominant disease, is caused by a triplet-repeat mutation, with
expansion (typically 40-55 repeats) of a CAG triplet. This gives rise to a long sequence of
glutamine residues in a protein of unknown function called huntington . Huntington's disease may
demonstrate genetic anticipation with an increasingly early onset in succeeding generations: this is
due to an increase in the number of triplet repeats. Huntington's disease is inevitably fatal.
The other types of mutation listed can all cause mutations in proteins as a result of the insertion of
the wrong amino acid (point and frame-shift mutations); incorporation of amino acid sequences
not normally present (splicing mutations) or premature or late termination of transcription
(termination mutations).
Which vector causes onchocerciasis?
freshwater snail
Simulium (black fly)
Ixodes tick
Chrysops (horse fly)
Sand fly
Your answer was CORRECT
Explanation
Some common vectors:
Vector Disease
Simulium (black fly) onchocerciasis
Chrysops (horse fly) loa loa
Freshwater snail schistosomiasis
Ixodes Lyme disease
Which skin lesion is classically associated with Reiter's syndrome?
pustular psoriasis
keratoderma blennorrhagicum
eczema
erythema chronicum migrans
Your answer was CORRECT
Explanation
Skin involvement is seen in 20% of Reiter's syndrome cases. The classic lesion is known as
keratoderma blennorrhagicum and commonly affects the palms and soles.
The following are typical of Sjogren's syndrome EXCEPT:
a lymphocytic cellular infiltrate
associated connective tissue disorder
involvement of the lacrimal gland and the oral mucosa
increase in the number of conjunctival goblet cells
Your answer was CORRECT
Explanation
In Sjogren's syndrome there is a lymphocytic cellular infiltrate affecting the acinar glands of the
conjunctiva, oral mucosa and lacrimal gland. There is resulting dry eye and dry mouth. It is
commonly associated with other connective tissue autoimmune diseases such as rheumatoid
arthritis. The number of conjunctival goblet cells is reduces.
The corneal findings in tyrosinaemia most closely resemble those of:
map-dot-fingerprint dystrophy
herpes simplex keratitis
Fabry's disease
Wilson's disease
Your answer was CORRECT
Explanation
In tyrosinaemia, elevated serum tyrosine levels lead to lysosomal instability with dermal and
ocular inflammation, as well as mental retardation. Non-staining pseudodendrites may recur and
be misdiagnosed as herpes simplex virus.
All of the following are risk factors for the development of cutaneous cancers EXCEPT:
greater natural skin pigmentation
increased age
increased sun exposure
red hair
Your answer was CORRECT
Explanation
Risk factors for cutaneous cancers include:
sun exposure
fair skin
family history
past history of skin cancer
fair and red hair
Naturally dark skin pigmentation is not associated with increased risk of cutaneous cancer.
All of the following are risk factors for the development of cutaneous cancers EXCEPT:
greater natural skin pigmentation
increased age
increased sun exposure
red hair
Your answer was CORRECT
Explanation
Risk factors for cutaneous cancers include:
sun exposure
fair skin
family history
past history of skin cancer
fair and red hair
Naturally dark skin pigmentation is not associated with increased risk of cutaneous cancer.
What type of abnormality is most likely to cause such a severe phenotype?
balanced autosomal translocation
sex chromosome aneuploidy
paracentric inversion
unbalanced autosomal translocation
pericentric inversion
Your answer was INCORRECT
Explanation
As a rule, the clinical effects of a chromosome abnormality reflect the amount of imbalance of
genetic material. For example, all autosomal monosomies and most autosomal trisomies are
incompatible with life, the exceptions being trisomy 13 (Patau's syndrome), trisomy 18 (Edward's
syndrome) and trisomy 21 (Down's syndrome): only the last of these carries a reasonable life
expectancy.
Inversions (where a piece of a chromosome is swapped around itself) often do not have a clinical
phenotype but are commonly found as an incidental finding. Occasionally the inversion
breakpoint is across a critical gene and causes a clinical disorder.
Balanced autosomal translocations are also not normally associated with morbidity. These may
only come to light when offspring are born with an unbalanced autosomal translocation that
results in an abnormal copy number for one or more chromosome regions (eg trisomic for one
region and monosomic for another). Such chromosome abnormalities are generally associated
with congenital abnormalities, relatively short stature and intellectual impairment, although the
exact phenotype can vary widely.
Sex chromosome aneuploidies are associated with comparatively less-severe phenotypes, eg
Klinefelter's syndrome (X-X-Y) and Turner's syndrome (X-O).
A biopsy is taken from the inferior fornix of a patient with suspected ocular cicatricial
pemphigoid. What findings on histological examination are most likely to confirm the disease?
hypersensitivity angiitis in sub-mucosal stroma
complement and immunoglobulin bound to epithelial basement membrane
intra-epithelial immunoglobulin and intraepithelial bullae
granulomatous destruction of epithelial basement membrane
Your answer was CORRECT
Explanation
Figure: Immunofluorescent microscopy of conjunctiva in OCP demonstrating linear
immunoglobulin deposition to epithelial basement membrane
OCP is characterised histologically by complement and immunoglobulin (IgG, IgA) bound to the
epithelial basement membrane.
Ocular cicatricial pemphigoid:
a chronic disease affecting mucosal surfaces
typically elderly population
usually bilateral (but may be asymmetric)
chronic conjunctivitis
dry eyes
sub-conjunctival fibrosis
decreased goblet cells
obstruction of ducts of the lacrimal and accessory lacrimal glands
symblepharon
shortening of the fornices
ankyloblepharon
keratinization of the conjunctival epithelium
corneal neovascularization and scarring
conjunctival scrapings show lymphocytes, plasma cells, and eosinophils
IgG, IgA at the level of the basement membrane on immunofluorescent stains
extraocular mucosal and skin lesions are common
differential:
o drugs including antivirals, miotics (both direct and indirect), epinephrine, and timolol
o chemical burns
o radiation treatment
o ocular rosacea
o Stevens-Johnson syndrome
o conjunctival carcinoma
Treatment:
o oral dapsone therapy
o systemic prednisone for acute exacerbations
o cyclophosphamide when treatment failure or intolerance develops with dapsone
Kaposi's sarcoma has been linked to:
cytomegalovirus
varicella zoster virus
human papillomavirus
pox virus
human herpes virus 8
Your answer was INCORRECT
Explanation
Kaposi's sarcoma is predisposed in immuno-suppressed individuals (e.g. AIDS). The tumor has
been associated with human herpes virus 8. No treatment is necessary for ocular lesions unless
they are bulky and unsightly. Doxorubicin has been used for systemic treatment. With immune
reconstitution, these lesions may regress.
Which one of the following regarding Lyme disease is FALSE?
recommended therapy includes either oral tetracycline or oral penicillin, with intravenous
antibiotics reserved for neurologic involvement or multiple recurrences
the most common eye finding is a chronic iridocyclitis with vitreous cells
the earliest eye finding is typically a follicular conjunctivitis
current serologic tests for Lyme disease are approaching >90% sensitivity
the other organ systems commonly affected by Lyme disease include skin, central nervous system,
cardiovascular, and musculoskeletal systems
Your answer was CORRECT
Explanation
Lyme immunofluorescent antibody titers and enzyme-linked immunosorbent assay (ELISA) for
IgM and IgG are positive in only 40% to 60% of cases.
A patient with epicanthic folds, myopia, single transverse palmar crease and keratoconus presents
to clinic. What iris anomaly are you most likely to find?
corectopia
mammillations
Koeppe nodule
Lisch nodules
Bruschfield spots
Your answer was INCORRECT
Explanation
The clinical features described above are consistent with Down's syndrome (trisomy 21), which is
associated with Bruschfield spots.
All are true of Reiter's syndrome EXCEPT:
it consists of a triad of urethritis, arthritis and conjunctivitis
ocular involvment usually follows urethritis
bacterial cultures are usually diagnostic
approximately 85% of patients are HLA-B27 positive
Your answer was CORRECT
Explanation
Reiter’s sydrome is characterised by the classic triad of urethritis, conjunctivitis, and arthritis.
Reiters syndrome is believed to be a post-infective phenomenon as patients frequently have a
history of foreign travel. However, bacterial cultures are frequently negative and do not aid in the
diagnosis or treatment.
Postulated causes of Reiter’s syndrome include:
Non-gonococcal urethritis
o Ureaplasma urealyticum
o Chlamydia trachomatis
Diarrheal illness
o Shigella
o Salmonella
o Yersinia
Other systemic and ocular features of Reiter's include:
keratoderma blennorrhagicum
iritis
keratitis
balanitis
prostatitis
cystitis
spondylitis
fasciitis
tendonitis
oral mucosal lesions
Which renal disorder is associated with uveitis especially in adolescent girls:
Alport syndrome
IgA glomerulonephritis
Goodpastures syndrome
Tubulointerstitial nephritis
Your answer was INCORRECT
Explanation
Both tubulointerstitial nephritis and IgA glomerulonephritis are known to be associated with
anterior uveitis. TINU (tubulointerstitial nephritis and uveitis) tends to affect adolescent girls with
renal disease preceding uveitis. IgA glomerulonephritis usually occurs in the third to fourth
decades.
In women, choroidal metastases most commonly derive from a primary tumor in the:
lung
breast
colon
ovarian
cervical
Your answer was CORRECT
Explanation
Choroidal metastases are most commonly from a primary lung tumor in men and from a primary
breast tumor in women.
All of the following conditions may present with signs and symptoms indistinguishable from
typical idiopathic inflammatory orbital disease (pseudotumour) EXCEPT:
polyarteritis nodosa
Churg-Strauss syndrome
Wegener's granulomatosis
SLE
sarcoidosis
Your answer was INCORRECT
Explanation
Sarcoidosis is generally not associated with pain and usually spares orbital soft tissues.
Parinaud’s oculoglandular syndrome is most likely to be caused by:
Brucella
Bartonella
Cytomegalovirus
Lyme
Your answer was CORRECT
Explanation
Parinaud’s oculoglandular syndrome is the combination of granulomatous conjunctivitis in one
eye and swollen ipsilateral pre-auricular lymph nodes. Most cases are caused by cat-scratch
disease, although it is an unusual feature of this condition. Occasionally, it may be caused by other
infections.
This question came in the 2014 FRCOphth.
All of the following are ocular manifestations of Wegener's EXCEPT:
scleritis
orbital mass
iritis
corneoscleral ulceration
Your answer was INCORRECT
Explanation
Wegener's can cause corneascleral ulceration, scleritis and an orbital mass. However, iritis is not
specifically associated with Wegener's granulomatosis.
Which of the following statements regarding acquired ocular syphilis is FALSE?
the end stage of retinal vasculitis and inflammation may resemble retinitis pigmentosa
treatment of syphilitic uveitis necessitates the same antibiotic regime as neurosyphilis
ocular findings may include iris nodules or vascularised papules
the eye can be involved in both secondary and tertiary syphilis
the FTA-Abs test is a useful measure of disease response to therapy
Your answer was INCORRECT
Explanation
Although non-treponemal tests such as the Venereal Disease Research Laboratory (VDRL) and
rapid plasma reagin (RPR) titers decrease with successful syphilis treatment, the fluorescent
treponemal antibody-absorption (FTA-ABS) titer usually does not decrease after treatment. Ocular
inflammation (e.g., uveitis, chorioretinitis) secondary to syphilis should be treated as
neurosyphilis. Other options supplied are true
What vitamin supplementation is required for a patient with xerophthalmia?
Vitamin E
Vitamin C
Vitamin B6
Vitamin D
Vitamin A
Your answer was CORRECT
Explanation
Xerophthalmia is caused by vitamin A deficiency, and it is a major cause of childhood blindness.
Ocular features of vitamin A deficiency include:
impaired dark adaptation
nyctalopia
dryness and thickening of conjunctiva and cornea
Bitot's spots, which are white plaques of keratinized epithelial cells
keratomalacia
infective keratitis
blindness
A 23 year old male complains of reduced vision. Systemic enquiry reveals he is under
investigation for progressive muscle weakness. On examination he has cataracts, an expressionless
face, frontal balding and gonadal atrophy.
How is this condition inherited:
autosomal recessive
autosomal dominant
X-linked recessive
mitochondrial
Your answer was CORRECT
Explanation
This is a case of myotonic dystrophy which is an autosomal dominant condition due to a mutation
in chromosome 19.
A patient has coarse dry skin, eyebrow thinning, weight gain, depression and cold intolerance.
What is the most likely diagnosis?
Graves disease
hypothyroidism
Cushing's disease
Addison's disease
Your answer was CORRECT
Explanation
The features described are strongly suggestive of hypothyroidism.
Hypothyroidism is associated with:
dry skin
eyebrow thinning
weight gain
depression
cold intolerance
peaches and cream skin complexion
hoarse voice
amenorrheoa
anaemia
hypercholesterolaemia
Posterior subcapsular cataract is a feature of:
NF-2 only
NF-1 only
both NF-1 and NF-2
neither NF-1 nor NF-2
Your answer was CORRECT
Explanation
NF-2 is associated with posterior subcapsular cataract. Bilateral acoustic neuromas is
characteristic of NF-2.
NF-1 is characterised by:
AD inheritence
cafe-au-lait patches
axial freckles (pathognomonic)
short stature
macrocephaly
facial hemi-atrophy
Lisch nodules
ON gliomas
meningiomas
plexiform neurofibromas
absence of greater wing of sphenoid
prominent corneal nerves
choroidal hamartomas
retinal tumours
o astrocytic hamartomas
o combined hamartomas of RPE and retina
Genomic imprinting is seen in which of the following conditions?
Huntington's chorea
Hurler's syndrome
Neurofibromatosis
Marfan's syndrome
Prader-Willi
Your answer was CORRECT
Explanation
The term 'genomic imprinting' refers to the dependence of phenotype on whether the gene deletion
is inherited from the mother or father. An example is Prader-Willi and Angelman syndromes.
Prader-Willi syndrome is a result of a paternal deletion of a gene on chromosome 15; Angelman
syndrome results if the same gene is deleted from the maternal side. In both syndromes, 3–5% of
cases are a result of uniparental disomy (both chromosomes from the same parent).
Regarding other options in the question above:
Neurofibromatosis is an autosomal-dominant disorder with 95% penetrance. The gene is located
on chromosome 17.
Huntington's chorea is a result of a triplet codon repeat. There is a 35–90 repeat segment of CAG
nucleotides on chromosome 4p 16.3. It is rare in childhood, presenting later in adult life with
dementia, chorea and rigidity.
Hurler's syndrome (type I) is an autosomal-recessive disorder occurring due to a defect in
chromosome 4p.
Marfan's syndrome is an autosomal-dominant disorder affecting chromosome 15q.
The mucopolysaccharidosis called Hunter's syndrome is inherited as:
X-linked recessive
X-linked dominant
autosomal dominant
autosomal recessive
Your answer was CORRECT
Explanation
All of the mucopolysaccharidoses are autosomal recessive disorders except type II (Hunter's)
which is X-linked recessive.
Which statement is FALSE regarding Toxocara infection:
eggs are passed in dog faeces
it is associated with an increase in eosinophil count
it can cause a rapid inflammation in the retinal mid-periphery
larvae when active produce an intense inflammatory response
it can cause a low-grade fibrous reaction in the retina
Your answer was CORRECT
Explanation
Toxocara laravae do not elicit an inflammatory response but when they die, the immune system
becomes activated which can cause one of the following in the eye:
low grade fibrous retinal involvement
rapid retinal inflammation with eosinophilia
vitritis and pars planitis
Which of the following is TRUE regarding mucous membrane pemphigoid?
males are more commonly affected than females
80% have skin lesions
histopathology shows linear deposition of IgG along the basement membrane zone
20% have mouth involvement
Your answer was CORRECT
Explanation
In MMP, the mouth is the most common site of involvement (80%), followed by the conjunctiva.
The skin is involved in approximately 20%. Females are more commonly affected (ratio 2:1).
Histology of lesions reveals linear deposition of IgG, C3, or less commonly IgA along the
basement membrane zone.
This question appeared in the 2014 FRCOphth Part 2.
A patient is sent to an ophthalmologist by a gastroenterologist to rule out Wilson's disease. The
most vital and sensitive aspect of the ophthalmologist's examination is:
gonioscopy
Amsler grid testing with red gratings
confocal imaging
tonometry
slit-lamp anterior segment examination
Your answer was CORRECT
Explanation
Figure: Gonioscopy showing golden brown deposit in Descemet's membrane consistent with
Kayser-Fleischer ring deposition
The earliest sign of copper deposition in Descemet's membrane (Kayser-Fleischer ring) is
detectable only at the far periphery with gonioscopy. Slit-lamp examination alone is insufficient.
Which of the following conditions are associated with the HLA-B27 genotype?
1. inflammatory bowel disease
2. Reiter's syndrome
3. ankylosing spondylitis
4. psoriatic arthritis
1 and 3
2 and 4
4 only
1, 2, and 3
1, 2, 3, and 4
Your answer was CORRECT
Explanation
All of the diseases listed, along with reactive arthritis are known as the seronegative
spondyloarthropathies, a group of autoimmune disease strongly associated with HLA-B27
positivity and acute anterior uveitis. The term seronegative refers to rheumatoid factor (RF);
patients with these diseases by definition do not have a positive RF.
A 25-year-old woman is referred to you for treatment of her asthma. She is otherwise well and
there is no significant previous medical history. During the consultation it transpires that her sister
died of cystic fibrosis and she is worried about having a child affected with the same disease. Her
chest X-ray is normal.
Assuming a population carrier frequency of 1 in 25, what is the chance of her having an affected
child?
1 in 200
1 in 150
1 in 100
1 in 10
1 in 50
Your answer was INCORRECT
Explanation
Your patient would have a 2 in 3 chance of being a carrier, as she is not affected herself (the
history of asthma is coincidental). Her partner would have a 1 in 25 chance of being a carrier. The
chance of having an affected child if both are carriers is 1 in 4. Therefore the overall chance is 2/3
x 1/25 x 1/4 = 1/150.
The commonest mistake (which also applies to other autosomal-recessive diseases) is to assume
that her chance of being a carrier is 1 in 2, on the premise that there is a 1 in 4 chance of being
affected, 1 in 4 of not being a carrier and a 1 in 2 chance of being a carrier. As she is not affected,
the chance of being a carrier is in fact 2 in 3 and the chance of not being a carrier is 1 in 3
The following stages are involved in leukocyte movement to a site of inflammation EXCEPT:
adhesion
division
migration
rolling
Your answer was INCORRECT
Explanation
Movement of leukocytes into the site of inflammation involve the following stages: rolling, firm
adhesion, tight adhesion, migration.
A patient has axillary freckles, prominent corneal nerves and absence of the greater wing of
sphenoid.
Which iris abnormality would you expect?
ectropion uveae
Koeppe nodule
corectopia
naevus
Bruschfield spots
Your answer was INCORRECT
Explanation
The clinical features described are consistent with neurofibromatosis-1. In terms of iris
abnormalities, NF-1 is associated with Lisch nodules, ectropion uveae and occasionally iris
mammillations.
Which is FALSE regarding Cat scratch disease:
it is caused by Bartonella henselae
the optic disc is the primary target
it is a viral-associated disorder
can be confirmed by indirect fluorescent antibody
Your answer was INCORRECT
Explanation
Cat scratch is caused by Bartonella henselae, which is a gram negative rod. Other options are true
A patient with Behcet's disease is commenced on azathioprine. All of the following are possible
side-effects of azathioprine EXCEPT:
gastrointestinal upset
bone marrow suppression
hepatotoxicity
tremor
Your answer was CORRECT
Explanation
Bone marrow suppression, hepatotoxicity and GI upset are all recognized side-effects of
azathioprine. Monitoring with FBC and LFTs is therefore necessary. Tremor is not a well-
recognised side-effect of azathioprine.
In which tissue does the Toxoplasma parasite survive best:
cerebral neurons
red blood cells
bronchial cells
hepatic cells
Your answer was CORRECT
Explanation
The brain and retina are the sites where Toxoplasma organisms survive best.
von Hippel-Lindau disease is most likely to be associated with:
capillary haemangioma of the retina
diffuse choroidal haemangioma
peripheral acquired haemangioma of the retina
astrocytic hamartoma of the retina
Your answer was CORRECT
Explanation
Figure: retinal capillary haemangioma.
von Hippel-Lindau presents with:
retinal capillary haemangiomas (which can cause exudative RD)
cerebellar haemangioblastomas
pancreatic, hepatic, and renal cysts
renal cell carcinoma
pheochromocytomas
All of the following viruses are associated with human carcinoma EXCEPT:
Hepatitis A
Epstein-Barr virus
Human Papilloma virus
Hepatitis C
HIV
Your answer was CORRECT
Explanation
Viruses that are associated with carcinoma include: Epstein-Barr virus (nasopharyngeal
carcinoma), hepatitis B and C (hepatocellular cancer), HIV virus (lymphoma, Kaposi's sarcoma),
human papilloma virus (cervical cancer).
Which of the following is NOT a common, visually-significant late complication in patients with
severe uveitis from juvenile idiopathic arthritis (JIA)?
choroidal neovascular membrane
cataract
secondary glaucoma
band keratopathy
Your answer was CORRECT
Explanation
In JIA patients with severe uveitis, the most common complications are:
band keratopathy (40%)
cataract (30%)
secondary glaucoma (15%)
CNVM is not a recognised complication of JIA-associated uveitis.
All of the following are features of homocystinuria EXCEPT:
lens subluxation
closed-angle glaucoma
osteoporosis
short stature
intravascular thrombosis with general anaesthesia
Your answer was CORRECT
Explanation
Homocystinuria is associated with tall stature. Other options above are true.
Homocystinuria is characterised by:
AR inheritance
tall stature
osteoporosis
scoliosis
chest deformities
mental retardation (50%)
increased risk of thrombotic vascular occlusions
lens dislocation (90%), generally inferior and bilateral
angle closure glaucoma if lens dislocates anteriorly
Which of the following best describes the ocular features of Addison's disease:
hyperpigmentation of conjunctiva and slow flow retinopathy
periorbital oedema, cataract and loss of lateral third of eyebrow
hyperpigmentation of the conjunctiva and lids with hypertensive retinopathy
hyperpigmentation of lids and conjunctiva, papilloedema and optic atrophy
Your answer was INCORRECT
Explanation
Addison's disease is the result of destruction of the adrenal cortex with resulting reduction in
glucocorticoids, mineralocorticoids and sex steroid production. Features include:
pigmentation of mouth, hands, face
hyperpigmentation of lids and conjunctiva
papilloedema
optic atrophy
Hunter's syndrome is caused by a deficiency of:
alpha lecithin cholesterol acyltransferase
beta glucuronidase
iduronate sulphate sulphatase
alpha L iduronidase
alpha galactosidase
Your answer was CORRECT
Explanation
Hunter's syndrome is caused by a deficiency of iduronate sulphate sulphatase.
The ocular effects are:
corneal clouding
pigmentary retinopathy
optic atrophy
The most common intraocular infection associated with human immunodeficiency virus (HIV)
infection is:
syphilitic uveitis
toxoplasmosis
cytomegalovirus (CMV) retinitis
pneumocystis choroiditis
acute retinal necrosis secondary to herpes simplex virus (HSV)
Your answer was CORRECT
Explanation
Cytomegalovirus (CMV) retinitis, along with Pneumocystis carinii pneumonia and Kaposi's
sarcoma, was one of the infections recognized early in the course of the epidemic as a defining
feature of the disease.
Which organism is transmitted by consumption of unpasteurized milk or uncooked meat?
Treponema pallidum
Bartonella henselae
Borrelia burgdorferi
Brucella melitensis
Your answer was CORRECT
Explanation
Transmission mechanisms:
Brucella - unpasteurised milk or uncooked meat (Brucellosis)
Bartonella - cat scratch or bite (Cat scratch disease)
Borrelia - tick bite (Lyme disease)
Treponema - sexual contact (Syphilis)
Toxoplasma - contact with cats and cat faeces
Toxocara - contact with dog faeces
In which of the following hereditary conditions will affected males have a significant risk of
fathering affected sons?
Duchenne's muscular dystrophy
myotonic dystrophy
cystic fibrosis
Haemophilia type A
MELAS (myopathy, encephalopathy, lactic acidosis, stroke)
Your answer was CORRECT
Explanation
Absence of male-to-male transmission is a hallmark of X-linked genetic conditions (such as
Duchenne's and haemophilia type A).
In mitochondrial diseases (e.g. MELAS) neither male nor female offspring of an affected male
will inherit as the mutated mitochondria are passed down in the female egg.
Cystic fibrosis (CF) is an autosomal-recessive condition, so, in theory, a male with CF could have
an affected son if his partner was a carrier. (The carrier frequency in northern Europeans is
approximately 1 in 25.) However, in practice, males with CF are invariably infertile as congenital
bilateral absence of the vas deferens (CBAVD) is a cardinal feature of CF.
Myotonic dystrophy is an autosomal-dominant condition and therefore there is a 50% risk to the
male and female offspring of an affected male, therefore this is the correct answer.
Which ocular condition is predisposed in a patient with homocystinuria:
dislocation of the lens
glaucoma
retinal macroaneurysm
angle recession
macular degeneration
Your answer was INCORRECT
Explanation
Patients with homocystinuria are more likely to suffer lens dislocation due to weakened zonular
fibres.
A 20-year-old man is referred for investigation of hypogonadism and infertility. He went through
normal puberty and there is no significant family history. On examination he is tall with
gynaecomastia and small testes. He has a normal sense of smell.
Which condition would be highest on your list of differential diagnoses?
Fragile X syndrome
Klinefelter's syndrome
Marfan's syndrome
XYY syndrome
Kallmann's syndrome
Your answer was CORRECT
Explanation
Klinefelter's is due to the addition of an extra X chromosome (X-X-Y). It is the most common sex
chromosome abnormality (1 in 600 male births) and the most common cause of male
hypogonadism and infertility. Affected males tend to be tall and can have mild learning
difficulties, although many have normal intellect. X-Y-Y, Fragile X and Marfan's syndromes are
not associated with hypogonadism or infertility. Fragile X syndrome is associated with macro-
orchidism and moderate to severe learning disability. Kallmann's syndrome is associated with
anosmia and hypogonadotrophic hypogonadism (as opposed to hypergonadotrophic
hypogonadism in Klinefelter's syndrome).
Which is FALSE regarding Human Immunodeficiency Virus (HIV):
it gains entry by binding to CD4 antigen on the cell surface
the earliest sign of infection is p24 protein in the blood
it affects CD4 positive T cells
Type II is primarily homosexually transmitted
Your answer was CORRECT
Explanation
Type I HIV occurs primarily in urban centres in the USA and Europe and is more common in
homosexuals and intravenous drug abusers. Type 2 is more common in Africa and is mainly
heterosexually transmitted.
Which antibiotic is most appropriate in the context of toxoplasma retinitis?
rifampicin
cefuroxime
metronidazole
azithromycin
Your answer was INCORRECT
Explanation
Toxoplasma is an obligate intracellular protozoan. Therapeutic options include: clindamycin,
azithromycin, sulfadiazine, pyrimethamine and co-trimoxazole.
Alport syndrome is caused by defective:
Type 1 collagen
fibrillin
elastin
Type 4 collagen
Type 2 collagen
Your answer was CORRECT
Explanation
Marfan's: defective fibrillin
Ehlers-Danlos: defective Type 1 (or Type 3) collagen
Stickler's: defective Type 2 (or Type 9) collagen
Alport's: defective Type 4 collagen
Which is FALSE regarding tuberculosis?
the Mantoux test is almost always positive in severe TB
the chest x-ray usually shows patchy shadowing in the upper zones of the lungs in pulmonary TB
primary TB is symptomless in the majority of patients
it is a notifiable disease in the UK
Your answer was INCORRECT
Explanation
The Mantoux test is negative in up to 50% of patients with severe TB.
Which is FALSE of ocular toxoplasmosis in acquired immunodeficiency syndrome (AIDS):
all patients should undergo brain MRI imaging
infection typically develops adjacent to an old toxoplasmosis scar
primary ocular toxoplasmosis is more common in patients with AIDS
it is more severe than toxoplasmosis in an immunocompetent patient
Your answer was INCORRECT
Explanation
Ocular toxoplasmosis in patients with AIDS rarely shows the typical fundus scar of a previous
infection. Primary ocular toxoplasmosis, associated with CNS toxoplasmosis, is more common in
patients with AIDS. Thus, all patients should undergo brain MRI.
The following are true about blood coagulation EXCEPT:
thrombin converts fibrinogen to fibrin
heparin inhibits blood coagulation through its interference with vitamin K metabolism
platelets are essential for blood clotting
vitamin K is responsible for the production of factors II, VII, IX and X
Your answer was CORRECT
Explanation
Heparin increases the activity of anti-thrombin III which inhibits the clotting cascade. All other
statements are true.
Which of the following conditions developing in a healthy man in his 20s or 30s should raise
suspicion of coincident human immunodeficiency virus (HIV) infection?
1. acute retinal necrosis
2. syphilitic uveitis
3. herpes simplex keratitis
4. herpes zoster ophthalmicus (HZO)
1 and 3
1, 2, and 3
4 only
1, 2, 3, and 4
2 and 4
Your answer was INCORRECT
Explanation
The development of herpes zoster ophthalmicus (HZO) in a young and otherwise healthy patient
should raise the suspicion of immunocompromise (leukaemia, chemotherapy, acquired
immunodeficiency syndrome). Sexually transmitted diseases tend to occur together; and this is
particularly true of syphilis and human immunodeficiency virus (HIV). If a clinician obtains
serologic studies for one disorder he or she should strongly consider testing for the other as well
(joint fluorescent treponemal antibody-absorption [FTA-ABS] and anti-HIV titers).
Regarding Type II Diabetes Mellitus:
the tissue response to insulin is normal
it is associated with certain HLA groups
there is a substantial loss of beta-cells in the islets of Langerhans
it is less common than type I diabetes mellitus
there is an increased risk amongst first-degree relatives
Your answer was INCORRECT
Explanation
Type II diabetes mellitus (DM) is more common than Type I DM. Unlike Type I DM, the beta-
cells in the islets of Langerhans are usually not lost in Type II DM. There is, however, resistance
of cellular response to insulin in Type II DM. There is an increased risk of Type II DM amongst
the first degree relatives of sufferers. Type I DM is associated with certain HLA groups but not
Type II DM.
A 30-year-old Japanese man presents with blurred vision. On examination, he has a hypopyon,
mild AC cells, and evidence of peri-phlebitis. He complains of arthritis, acne on his face and back
and has red plaques on his shins.
Which statement concerning the therapeutic treatment of this condition is TRUE?
anti-TNF agents are useful in the acute stages and should be tapered rapidly
periocular steroids alone are usually effective in preventing a relapse of ocular inflammation
cyclosporine may be useful in this condition, but liver toxicity is a common limitation
colchicine may be helpful in preventing recurrences
oral prednisolone is usually effective in preventing a relapse of ocular inflammation
Your answer was CORRECT
Explanation
This patient has Behcet's disease. Commonly used drugs in this condition include anti-TNF agents
(e.g. etanercept or infliximab), systemic corticosteroids, interferon-alpha, cyclosporine,
azathioprine and colchicine.
While systemic corticosteroids may be initially effective in treating the ocular inflammation in
Behcet's they do not prevent long-term progression.
For ocular disease, anti-TNF agents are gaining widespread use. Azathioprine has also been
widely accepted as an initial treatment choice. For severe eye disease (significant drop in visual
acuity, retinal vasculitis, or macular involvement), either cyclosporine A or infliximab (anti-TNF)
may be used in combination with azathioprine and corticosteroids.
Cyclosporine is mainly associated with renal (not liver) toxicity.
Colchicine, although becoming a more dated choice compared to biologics, has been shown to be
useful in reducing recurrences.
In men, choroidal metastases most commonly derive from a primary tumour of which organ:
colon
testicular
prostate
lung
breast
Your answer was CORRECT
Explanation
Choroidal metastases are most commonly from a primary lung tumor in men and from a primary
breast tumor in women.
Which is FALSE regarding Human Immunodeficiency Virus (HIV):
the earliest sign of infection is p24 core protein in the blood
the diagnosis of AIDS cannot be made on the basis of T lymphocyte counts
antibodies to HIV have no real protective or neutralizing function
infection can be diagnosed by detection of antibodies using ELISA
Your answer was CORRECT
Explanation
The diagnosis of AIDS can be made both on clinical grounds by the manifestation of indicator
diseases or it can be made on the basis of T cell count where CD4 T cell count falls below 200 per
microliter
Which condition causes a net-like abdominal rash?
polyarteritis nodosa
syphilis
Wegener's granulomatosis
sarcoidosis
Your answer was CORRECT
Explanation
Figure: Livido reticularis in a patient with polyarteritis nodosa.
Livdo reticularis consists of a mottled reticulated vascular pattern that appears as a lace-like
purplish discoloration of the skin, often on the abdomen. It is caused by obstruction of capillaries
with thrombi. It may be caused by a number of conditions including connective tissue diseases
such as polyarteritis nodosa, SLE, dermatomyositis and rheumatoid arthritis.
A patient has conjunctival and retinal telangiectasis. These findings are most consistent with
which systemic disease or syndrome?
Alport's syndrome
Fabry's disease
Refsum's disease
Lowe's syndrome
incontinentia pigmenti
Your answer was CORRECT
Explanation
Fabry's disease is an X-linked recessive condition caused by alpha galactosidase deficiency.
It is characterised by:
conjunctival and retinal vessel tortuosity/telangiectasis
cataract: either an anterior capsular/subcapsular or a radial posterior subcapsular cataract
vortex keratopathy
ocular motility abnormalities
All are true of ankylosing spondylitis EXCEPT:
90% of patients are HLA-B27 positive
conjunctivitis and scleritis are both recognised associations
it is more common in males
acute anterior uveitis is common and usually presents bilaterally
Your answer was CORRECT
Explanation
Acute anterior uveitis is common (affecting about 25% of AS patients) but it is usually unilateral.
It can affect each eye at different times but simultaneous involvement is rare.
What is the most likely diagnosis in a 30-year-old man with tall stature, gynaecomastia and
azoospermia?
XYY syndrome
Homocystinuria
Cystic fibrosis
Marfan's syndrome
Klinefelter's syndrome
Your answer was CORRECT
Explanation
Tall stature is a feature of homocystinuria, Klinefelter syndrome, Marfan's syndrome and XYY
syndrome. The combination of tall stature, gynaecomastia and infertility due to azoospermia is
diagnostic of Klinefelter's syndrome. Almost all males with Klinefelter syndrome are
azoospermic. Other features of this condition include reduced facial hair, tendency to obesity, and
small atrophic testes. Mild learning difficulties can be seen in some patients with Klinefelter's
syndrome but mental retardation is not a feature of this condition. Chromosome analysis will show
47 chromosomes, with two X and one Y chromosome (47, X-X-Y).
Patients with homocystinuria have tall stature, learning difficulties, lens dislocation, osteoporosis
and recurrent arterial thrombosis.
Features of Marfan's syndrome include tall stature, arachnodactyly, scoliosis, joint laxity, lens
dislocation, aortic root dilatation, dural ectasia, skin striae, and recurrent pneumothorax.
The XYY syndrome is characterised by tall stature, mild learning difficulties and behavioural
problems. Most males with this condition have normal fertility, although a small proportion of
patients will have infertility due to azoospermia.
A 29-year-old man presents with photophobia, pain in his wrists and feet, pain on urination, and
aphthous ulcers. Ocular examination reveals a mucoid conjunctival discharge and a mild cellular
reaction in the anterior chamber.
All of the following are consistent with the diagnosis EXCEPT?
Prostatitis
Recent history of diarrhea
Balanitis
Rash on the palms
Positive rheumatoid factor
Your answer was CORRECT
Explanation
This patient has Reiter's sydrome, characterised by the classic triad of urethritis, conjunctivitis,
and arthritis.
Other features of Reiter's include:
keratoderma blennorrhagicum
iritis
keratitis
balanitis
prostatitis
cystitis
spondylitis
fasciitis
tendonitis
oral mucosal lesion
HLA-B27 genotype (75%-90% of cases)
Causes of Reiter's syndrome include:
Non-gonococcal urethritis
o Ureaplasma urealyticum
o Chlamydia trachomatis
Diarrheal illness
o Shigella
o Salmonella
o Yersinia
Which of the following could NOT be caused by non-disjunction of chromosomes during cell
division?
Marfan's Syndrome
Down's Syndrome
Turner's Syndrome
Edward's Syndrome
Klinefelter's Syndrome
Your answer was CORRECT
Explanation
Non-disjunction is a failure of homologous chromosomes or chromatids to separate during mitosis
or meiosis. It can lead to an aneuploid (i.e. abnormal) number of chromosomes, such as in trisomy
(presence of a third chromosome) or monosomy (absence of one member of a chromosome pair).
Down's is characterised by trisomy of chromosome 21.
Edward's syndrome is caused by trisomy of chromosome 18.
Turner's syndrome has the genotype XO.
Klinefelter's is X-X-Y.
Marfan's syndrome is characterised by a defect in the gene Fibrillin-1 and is autosomal dominant.
A 20-year-old female with cystic fibrosis presents in early pregnancy wanting advice. Genetic
analysis reveals that her partner is a carrier of the cystic fibrosis gene.
What is the chance of her child having cystic fibrosis:
50%
75%
25%
100%
10%
Your answer was CORRECT
Explanation
The patient has cystic fibrosis and the husband is the carrier; therefore, as cystic fibrosis is
inherited autosomal recessively, the chances that her child will have cystic fibrosis are as high as
50%.
Which histology finding is most in keeping with sarcoidosis?
multi-nucleated giant cells
caseating granuloma
Langerhans cells
an outer rim of mostly killer T cells
Your answer was CORRECT
Explanation
Sarcoidosis is characterized by non-caseating granulomas. Within a granuloma, there is a central
collection of modified mononuclear phagocytes called epithelioid cells. Epithelioid cells may
conglomerate to form multi-nucleated giant cells known as Langhan's cells (not Langerhans cells).
The central epithelioid and Langhan's giant cells are surrounded by a rim of lymphocytes, mostly
T helper cells.
This question came in the 2014 FRCOphth.
A 19-year-old woman presents for a routine ophthalmic examination. Slit-lamp examination
discloses a vortex keratopathy bilaterally with telangiectatic conjunctival vessels. Her mother and
father are both healthy.
Which one of the following statements is FALSE?
other slit-lamp findings might include subtle posterior subcapsular cataract
a thorough drug history should be taken
fundus findings might include telangiectatic retinal vessels
the patient should be warned of potentially lethal renal failure
Your answer was INCORRECT
Explanation
The history is consistent with Fabry disease. Males develop renal impairment but not females.
Drugs may cause vortex keratopathy so it is important to take a thorough drug history.
Fabry disease is characterised by:
X-linked recessive inheritance **
Deficiency in alpha-galactosidase enzyme
Peripheral neuropathy
GI symptoms
Cardiac impairment
renal impairment in males (not females)
Skin: widespread angiokeratomas
Ocular:
o vortex keratopathy
o cataract
o retinal vessel tortuosity
** Note: an unusual feature of Fabry's disease is the presence of clinical signs and symptoms in
the majority of heterozygous female carriers of the condition, although these manifestations are
usually less severe and of later onset than in affected homozygous males
You review a 64-year-old man who is on warfarin therapy for recurrent atrial fibrillation. He
presents to the Accident and Emergency Department with bruising. An international normalised
ration (INR) check reveals that his INR is raised, at 6.5.
Which of the following drugs may result in increased INR when co-prescribed with warfarin
therapy?
carbamazepine
ciprofloxacin
phenobarbitone
primidone
rifampicin
Your answer was INCORRECT
Explanation
All of the other options above are hepatic enzyme inducers and result in accelerated metabolism of
warfarin and therefore reduced effect and decreased international normalised ratio (INR).
Chloramphenicol, ciprofloxacin, clarithromycin, erythromycin, metronidazole, nalidixic acid,
norfloxacin, ofloxacin, and sulphonamides are all associated with reduced warfarin metabolism
and enhanced anti-coagulant effect.
Which mediator of acute inflammation is NOT derived from cells:
histamine
complement
leukotriene
cytokines
Your answer was INCORRECT
Explanation
Most mediators are derived from cells with the exception of: complement, kinin, clotting factors,
and fibrinogen, which are all derived from the plasma.
The most important component of long-term therapy in a young man who is HLA-B27 positive is:
annual tonometry
oral non-steroidal antiinflammatory therapy
recurrent annual visual acuity testing
physical therapy
Your answer was INCORRECT
Explanation
Asymptomatic sacroiliac disease can be seen in patients with HLA-B27 spondylitis, particularly in
young men. Because irreversible damage may occur before the onset of significant symptoms and
simple physical therapy is effective in limiting disability, physical therapy, consisting of back
flexibility and stretching exercises, is recommended in young men who are found to be HLA-B27
positive.
Chronic sinopulmonary infections may be seen as part of:
tuberous sclerosis
neurofibromatosis
ataxia-telangiectasia
angiomatosis retinae
Your answer was CORRECT
Explanation
Patients with ataxia-telangiectasia may have associated thymic hypoplasia, defective T-cell
function and IgA deficiency, with severe respiratory infections.
Louis-Bar, or ataxia-telangiectasia is characterised by:
autosomal recessive inheritance
cerebellar dysfunction: ataxia, oculomotor apraxia, nystagmus
telangiectasia of conjunctiva and the skin of nose, ears, elbows and knees
delayed motor development
immunodeficiency including thymic aplasia: recurrent respiratory infections
premature graying of hair
cafe-au-lait spots
susceptability to cancer: lymphoma and leukaemia
During wound healing all are true EXCEPT:
granulation tissue is laid down in the early stage of healing
macrophages and lymphocytes are essential at the late stage of healing
healing by first intention occurs in a clean and closely approximated wound
the tensile strength of the scar is determined by the amount of collagen in the wound
Your answer was INCORRECT
Explanation
Macrophages have a role in wound healing, and an intact immune system is certainly important
for wound healing but lymphocytes cannot be said to play a central or essential role in wound
healing.
From which organ does Pneumocystis carinii retinitis usually disseminate in patients with AIDS?
CNS
lungs
nasopharynx
gastrointestinal tract
Your answer was INCORRECT
Explanation
Pneumocystis carinii occurs with CD4 counts less than 200. The differential includes
cytomegalovirus, Cryptococcus, fungi, or other protozoa such as Microsporidia. It usually
originates from the lungs. Treatment includes trimethoprim/sulfamethoxazole, atovaquone,
pentamidine, or dapsone.
Which of the following genetic conditions is NOT clearly linked with an increased risk of
neoplasms:
Chromosome 13q deletion
Down's syndrome
Neurofibromatosis Type 1
Turner's syndrome
Neurofibromatosis Type 2
Your answer was INCORRECT
Explanation
Turner's syndrome has not been convincingly linked with an increased risk of neoplasms.
Chromosome 13q deletion is associated with retinoblastoma. Neurofibromatosis Type 1 is
associated with meningioma, glioma and phaeochromocytoma. Neurofibromatosis Type 2 is
associated with bilateral acoustic neuroma. Down's syndrome is associated with acute leukaemia.
Which virus is NOT known to be associated with human cancer:
Human Immunodeficiency Virus
Epstein-Barr Virus
Human Papilloma Virus
Herpes Zoster Virus
Your answer was CORRECT
Explanation
Viruses associated with human cancer:
Epstein-Barr virus: Burkitt's lymphoma and nasopharyngeal carcinoma
Human Papilloma virus: cervical cancer
Hepatitis B and C: hepatic cancer
Human Immunodeficiency Virus: Kaposi's sarcoma and non-Hodgkin's lymphoma
Erythema nodosum is a feature of all EXCEPT:
Behcet's disease
Sarcoidosis
Wegener's granulomatosis
Crohn's disease
Hepatitis C
Your answer was INCORRECT
Explanation
Figure: Erythema nodosum on the shins of a patient after streptococcal pharyngitis.
Wegener's granulomatosis can cause pyoderma gangrenosum, but it is not usually associated with
erythema nodosum.
Erythema nodosum is caused by:
Infections
o streptococcus
o primary TB infections
o Hepatitis C
o EBV
o Cat scratch
o mycoplasma pneumoniae
o histoplasma
Automimmune
o Inflammatory Bowel disease
o Behcet's disease
o Sarcoidosis
Medications
o Sulfonamides
o Penicillins
o oral contraceptive
Cancer
o Non-Hodkins lympohma
o Pancreatic cancer
Pyoderma gangrenosum can be caused by:
inflammatory bowel including ulcerative colitis and Crohns
rheumatoid arthritis
seronegative arthritis
haematologic disease including leukaemia and myeloma
Wegener's granulomatosis
What is the most likely mode of spread of basal cell carcinoma?
local invasion
lymphatic
via the CSF
haematogenous
Your answer was CORRECT
Explanation
Basal cell carcinoma has low malignant potential. It rarely metastasizes via haematogenous or
lymphatic routes and most commonly is locally invasive. Squamous cell carcinoma, by contrast,
spreads most commonly via the lymphatics.
This question appeared in the 2014 FRCOphth Part 2.
What is the most likely mode of spread of basal cell carcinoma?
local invasion
lymphatic
via the CSF
haematogenous
Your answer was CORRECT
Explanation
Basal cell carcinoma has low malignant potential. It rarely metastasizes via haematogenous or
lymphatic routes and most commonly is locally invasive. Squamous cell carcinoma, by contrast,
spreads most commonly via the lymphatics.
This question appeared in the 2014 FRCOphth Part 2.
The classification of scleritis which has the strongest association with rheumatoid arthritis is:
necrotizing scleritis with inflammation
scleromalacia perforans
diffuse anterior scleritis
posterior scleritis
nodular anterior scleritis
Your answer was CORRECT
Explanation
Figure: Scleromalacia perforans
Patients with scleromalacia perforans (necrotizing scleritis without signs of inflammation) often
have long-standing rheumatoid arthritis.
Which of the following is an ocular association of Alport's syndrome?
retinitis pigmentosa
optic neuropathy
flecks on the retina
juvenile glaucoma
high myopia
Your answer was CORRECT
Explanation
Alport's syndrome can be inherited in both X-linked recessive (80%) and X-linked dominant (5%)
forms. Where X-linked recessive, heterozygous females may still experience haematuria - a
separate condition termed 'thin basement membrane nephropathy.' Reference: NIH National
Library of Medicine.
Alport's is caused by a defect in Type 4 (basement membrane) collagen. Clinically, it causes:
glomerular basement membrane nephritis
anterior lenticonus
cortical cataract
posterior polymorphous dystrophy
dot-fleck retinopathy (macular and mid-peripheral)
corneal dystrophies e.g. keratoconus
microcornea
X-linked dominant conditions are rare and include:
Aicardi syndrome
Alport syndrome (5% of cases)
Incontinentia pigmentii
This question came in the FRCS (Glasgow) Part 2 exam in October 2014.
Ocular toxoplasmosis is most common in which of the following countries:
Brazil
France
Japan
United States
Your answer was CORRECT
Explanation
Figure: Macular toxoplasmosis scars with re-activation at the superior edge.
Toxoplasma gondii is a ubiquitous intracellular parasite. It appears to be more prevalent in hot and
humid climates. The prevalence of infection around the world depends on climate, hygiene and
eating habits. Ocular toxoplasmosis is most common in Brazil, where its incidence is as high as
18% - likely due to the common practice of eating raw pork which may be contaminated with
Toxoplasma cysts
All of the following are features of hypertensive retinopathy EXCEPT:
lymphocytic infiltration of vessel walls
hyalinization
retinal hemorrhages
hard exudation
fibrinoid necrosis
Your answer was CORRECT
Explanation
Hypertensive retinopathy results in hyalinization of blood vessel walls. Further vessel narrowing
and ischaemia lead to retinal hemorrhages, fibrinoid necrosis of the vessels with exudation and
papilloedema. Lymphocytic infiltration is not a feature of hypertensive retinopathy.
A 40-year-old patient with a chronic history of sinusitis presents with a 10-day history of
worsening right-sided painful proptosis and chemosis. There is severe limitation of right eye
movements in all directions and a right RAPD. Slit lamp examination reveals sclerokeratitis.
The findings are most in keeping with:
mucormycosis
eosinophilic granuloma
orbital pseudotumour
Wegener's granulomatosis
Your answer was CORRECT
Explanation
The features of sclerokeratitis, orbital disease and co-existing sinus disease raise suspicion of
Wegener's. Mucormycosis can also arise from the sinuses but it occurs in immunocompromised
patients and is not associated with sclerokeratitis. 80% of Wegener's have a positive c-ANCA.
Treatment is with high dose steroids and other immunosuppressives such as cyclophosphamide.
Which statement regarding neurofibromatosis-1 is FALSE?
up to 50% of patients with plexiform neurofibromas involving the upper eyelid develop ipsilateral
glaucoma
cafe-au-lait spots appear in over 90% of patients with this disorder
tumors of the CNS, including optic nerve gliomas, astrocytomas, meningiomas, and
neurofibromas occur in 5% to 10% of patients with neurofibromatosis
Lisch nodules appear in over 90% of patients over the age of 6 years but are non-diagnostic
because they may appear in normal patients as well
Your answer was CORRECT
Explanation
Lisch nodules occur in 90% of NF-1 patients but they do NOT occur in normal patients.
Bruschfield spots (associated with Down's syndrome) and iris mamillations (associated with
neurofibromatosis and oculodermal melanocytosis) can be found in the normal population.
NF-1 is characterised by:
AD inheritence
café-au-lait patches
axial freckles (pathognomonic)
short stature
macrocephaly
facial hemi-atrophy
Lisch nodules
ON gliomas
meningiomas
plexiform neurofibromas
choroidal hamartomas (present in up to 100% of patients, many detectable only on SLO)
retinal tumours
o astrocytic hamartomas
o combined hamartomas of RPE and retina
absence of greater wing of sphenoid
prominent corneal nerves
A patient presents with blepharospasm, irritated eyes and gradual reduced vision. On examination,
you discover conjunctivitis, keratitis, polychromatic cataracts and disc swelling. You suspect this
presentation is related to the patient's background endocrine abnormality.
What is the most likely diagnosis?
hyperthyroidism
Addison's
hypoparathyroidism
hypothyroidism
hyperparathyroidism
Your answer was CORRECT
Explanation
Hypoparathyroidism is typically seen after throidectomy or parathyroidectomy. When idiopathic,
it may be associated with other autoimmune diseases such as vitiligo and Type 1 DM. The
condition causes hypocalcaemia which causes a range of systemic sequelae.
The ocular features include:
conjunctivitis
keratitis
polychromatic cataracts
papilloedema
CSF values can range from normal to markedly elevated
All are true of Behcet's disease EXCEPT:
a positive pathergy test is a criterion for diagnosis
ocular involvement is usually bilateral
ocular involvement occurs in over 90% of males
ocular and systemic features usually develop simultaneously
Your answer was CORRECT
Explanation
Behcet's disease is a multisystem disease characterized by recurrent episodes of orogenital
ulceration and vasculitis. It is rare for ocular and systemic features to develop simultaneously in
Behcet's disease. Usually systemic features occur first, and ocular involvement occurs within 2
years of oral ulceration. Ocular involvement occurs in about 95% of males and 70% of females; it
is usually bilateral.
All of the following are typical of anaphylactic shock EXCEPT:
bradycardia
rhinitis
hypotension
vomiting
Your answer was CORRECT
Explanation
Tachycardia (not bradycardia) is typical of anaphylactic shock. Bradycardia is a feature of
vasovagal syncope.
Ocular manifestations of Reiter's syndrome include all EXCEPT:
dacroadenitis
episcleritis
conjunctivitis
uveitis
Your answer was CORRECT
Explanation
Ocular manifestations of Reiter's include:
uveitis
conjunctivitis
episcleritis
scleritis
nummular keratitis
intermediate uveitis
papillitis
retinal oedema
retinal vasculitis
Corneal clouding is NOT seen in which of the following mucopolysaccharidoses (MPS)?
Type III (Sanfilippo's syndrome)
Type I-S (Scheie's syndrome)
Type I-H (Hurler's syndrome)
Type IV (Morquio's syndrome)
Your answer was CORRECT
Explanation
Corneal clouding is common in mucopolysaccharidoses (MPS) types I-H (Hurler's), I-S (Scheie's),
IV (Morquio's), and VI (Maroteaux-Lamy). It may rarely be seen in type II (Hunter's) but is never
seen in type III (Sanfilippo's).
What is the most commonly used steroid-sparing agent in the treatment of uveitis associated with
juvenile idiopathic arthritis (JIA)?
TNF-alpha
azathioprine
methotrexate
tacrolimus
Your answer was INCORRECT
Explanation
Methotrexate is the most commonly used steroid-sparing agent in the treatment of JIA and its
secondary uveitis.
Which one of the following regarding congenital syphilis is TRUE?
serologic testing for suspected congenital syphilis is generally not rewarding
syphilitic IK is a direct manifestation of active corneal infection
a diagnosis of syphilitic IK mandates a full course of treatment for neurosyphilis
the interstitial keratitis (IK) of congenital syphilis is generally bilateral and asymptomatic
Your answer was CORRECT
Explanation
Interstitial keratitis (IK) usually produces intense pain and photophobia. The immune response in
IK is felt to be an immune response to treponemal antigens (and not live organisms). Standard
regimens for neurosyphilis are sufficient to treat luetic IK. Although results of the rapid plasma
reagin (RPR) and Venereal Disease Research Laboratory (VDRL) tests may be negative in
congenital syphilis, those of the fluorescent treponemal antibody-absorption (FTA-ABS) are
usually positive.
Which of the following are common haematologic findings in patients with acquired
immunodeficiency syndrome (AIDS)?
1. CD4 lymphocytopenia
2. hypergammaglobulinemia
3. increased suppressor T cells, relative to helper T cells
4. granulocytopenia
2 and 4
1, 2, 3, and 4
4 only
1, 2, and 3
1 and 3
Your answer was CORRECT
Explanation
Patients with acquired immunodeficiency syndrome (AIDS) exhibit absolute lymphocytopenia,
elevated immunoglobulin (especially IgA and IgG), and increased suppressor T-cell counts,
particularly relative to helper T cells. Although there is an absolute lymphocytopenia, there may
be no leukocytopenia or granulocytopenia.
What is the predominant cause for damage to ocular structures from infection by Onchocerca?
infiltration of retinal tissue
toxins produced by the larvae
inflammatory reaction to dead microfilaria
obstruction of the trabecular meshwork
Your answer was CORRECT
Explanation
Onchocerca volvulus infection results in widespread dissemination of the microfilarial larvae.
These microfilaria can be seen swimming in the anterior chamber. The live organisms may cause a
mild uveitis and obstruct the trabecular meshwork; however, dead organisms incite a vigorous
inflammatory reaction, which causes much more ocular damage.
Which of the following syndromes is characterised by an anti-mongoloid slant of the palpebral
fissure:
Pfeiffer's
Down's syndrome
Treacher Collins
blepharophimosis
Your answer was CORRECT
Explanation
Figure: Treacher Collins syndrome
Treacher Collins is characterised by:
autosomal dominant
clefting syndrome
hypoplasia of the mid-face
pseudocolobomas of the eyelids
downward angle to the lateral canthi
micrognothia
dental anomalies
ear anomalies: small and malformed with conductive hearing loss
Cogan's syndrome is frequently associated with which systemic disorder?
systemic lupus erythematosus
polyarteritis nodosa
Wegener's granulomatosis
rheumatoid arthritis
Your answer was CORRECT
Explanation
Typical Cogan's syndrome is defined by non-syphilitic interstitial keratitis associated with
audiovestibular involvement similar to that of Meniere's disease with progressive hearing loss to
complete deafness within 2 years. The syndrome primarily affects young adults and involves other
organ systems in two-thirds of cases and gives a clinical picture reminiscent of systemic vasculitis
in one third of patients. Prognosis is dominated by the risk of deafness and by cardiovascular
complications, particularly the risk of aortitis with dissecting aneurysms. Treatment is with
corticosteroids. Cogan's has been frequently associated with polyarteritis nodosa.
Lyme disease is associated with which dermatologic finding:
erythema nodosum
folliculitis
erythema granulosum
erythema chronicum migrans
pyogenic granuloma
Your answer was CORRECT
Explanation
Lyme disease is associated with erythema chronicum migrans.
Neurofibromatosis is most likely to be associated with:
capillary haemangioma of the retina
diffuse choroidal haemangioma
peripheral acquired haemangioma of the retina
astrocytic hamartoma of the retina
Your answer was CORRECT
Explanation
Figure: retinal astrocytoma
NF-1 is characterised by:
AD inheritence
cafe-au-lait patches
axial freckles (pathognomonic)
short stature
macrocephaly
facial hemi-atrophy
Lisch nodules
ON gliomas
meningiomas
plexiform neurofibromas
absence of greater wing of sphenoid
prominent corneal nerves
choroidal hamartomas
retinal tumours
o astrocytic hamartomas (shown in the image above)
o combined hamartomas of RPE and retina
Which statement regarding homocystinuria is TRUE?
most patients will develop ectopia lentis
the primary defect leading to lens dislocation is an abnormal lens size
there is an association with coagulopathy causing bleeding diatheses
the disorder is inherited on an autosomal dominant basis
Your answer was CORRECT
Explanation
Homocystinuria is characterised by
autosomal recessive inheritance
inborn error of methionine metabolism
elevated serum methionine and homocystine
Marfanoid habitus
tall stature
fair hair: blond or red
malar flush
osteoporosis
scoliosis
increased risk of thrombo-embolic events
mental retardation (50%)
ectopia lentis (90%): usually downwards, absent zonules and accomodation
treatment: diet low in methionine and high in cysteine
What is the second most common method of human immunodeficiency virus (HIV) transmission
in Western countries?
blood transfusions
transplacental transmission
sexual intercourse
intravenous drug use
Your answer was CORRECT
Explanation
Intravenous drug use accounts for approximately 25% of cases of human immunodeficiency virus
(HIV) transmission. Sexual intercourse accounts for 70% of cases.
Findings consistent with Reiter's syndrome include all of the following EXCEPT:
acute non-granulomatous iridocyclitis
non-erosive oligoarthritis
severe retinal vasculitis
keratoderma blennorrhagicum
papillary conjunctivitis with corneal neovascularization
Your answer was CORRECT
Explanation
Figure: Keratoderma blennorrhagicum
While posterior uveitis may occur in Reiter's syndrome, including cystoid macular oedema, severe
retinal vasculitis is not a feature of this syndrome.
A 22-year-old Chinese patient reports decreased vision in both his eyes. On examination, there is a
small hypopyon right eye and a moderate cellular reaction left eye. Fundus examination shows
extensive periphlebitis and peri-arteritis bilaterally. On further questioning, you elicit a history of
arthritis of his knees and wrists, and painful lesions in his mouth and around his genitals.
Skin lesions common to this condition include all of the following EXCEPT:
thrombophlebitis
erythema nodosum
psoriasis
acne-like lesions over the back and face
Your answer was CORRECT
Explanation
The clinical features suggest Behcet's disease.
Skin lesions associated with Behcet's include:
erythema nodosum
typical acne-like lesions
folliculitis
thrombophlebitis
cutaneous hypersensitivity
The behcetine skin test (skin pathergy test) is performed by puncturing the skin with an empty
hypodermic needle. A positive test is indicated by the formation of a pustule at the puncture site
within minutes.
All of the following are ocular features of Trisomy 21 EXCEPT:
myopia
corneal marginal ulcers
cataract
Brushfield spots
epicanthal folds
Your answer was CORRECT
Explanation
The ocular features of Down's syndrome include:
keratoconus
cataract
myopia and hyperopia
squint
Brushfield spots
epicanthal folds
Which is FALSE regarding systemic lupus erythematosus:
anti-double stranded DNA bodies are raised
a small vessel vasculitis may occur
phospholipid antibodies are raised
it is a multisystem autoimmune condition
ocular involvement is common
Your answer was CORRECT
Explanation
Ocular involvement with SLE is rare. When present it can produce:
retinal microinfarcts
central retinal artery occlusion
central retinal vein occlusion
choroidopathy
Which condition is LEAST likely to be caused by underlying rheumatoid arthritis?
scleritis
iritis
scleromalacia perforans
peripheral corneal ulceration
Your answer was CORRECT
Explanation
Rheumatoid arthritis (RA) causes an immune-complex vasculitis, which impairs the circulation to
the anterior segment of the eye. It is a common cause of the following conditions: peripheral
corneal ulceration, scleritis and thinning of the sclera (scleromalacia perforans). While RA can
cause iritis, it is a very rare association, accounting for about 1% of all iritis cases.
You review a 28-year-old man with a family history of early thyroid carcinoma and
phaeochromocytoma. Your patient has been searching on the internet and has found information
about the RET proto-oncogene.
The proto-oncogene RET causes which thyroid cancer?
lymphoma
follicular
anaplastic
papillary
medullary
Your answer was CORRECT
Explanation
Mutations of the RET proto-oncogene on chromosome 10 are associated with multiple endocrine
neoplasia (MEN) 2a and 2b. MEN2a is associated with phaeochromocytoma, Cushing's syndrome,
medullary thyroid carcinoma, and parathyroid hyperplasia. MEN2b is similar, but patients have a
marfanoid habitus and intestinal and visceral ganglioneuromas, but not hyperparathyroidism.
Rarely, it may occur in isolation without other tumours. Screening for medullary thyroid
carcinoma is with the pentagastrin and the calcium infusion test with measurement of calcitonin.
Where there is a family history of MEN2, total thyroidectomy in childhood to prevent the
development of carcinoma is usually recommended. Prior to consideration of thyroidectomy,
screening for co-existent phaeochromocytoma is essential, so that alpha-blockade can be
considered, otherwise the risk of on-table hypertensive crisis is high.
The critical difference between primary and secondary Sjogren's syndrome is:
salivary gland involvement in the primary group
the presence of anti-Ro auto-antibodies in the primary group
HLA-Dw3 subtype in the primary group
association with systemic connective tissue disease in the secondary group
Your answer was CORRECT
Explanation
The definition of secondary Sjogren's syndrome is sicca complex plus co-existent autoimmune
disease. Anti-Ro (SS-A and SS-B) autoantibodies may be more prevalent and at higher
concentrations in primary disease (no systemic disorder), but this is not a reliable differentiating
feature.
Which one of the following concerning the manifestations of sarcoidosis is FALSE?
the most commonly involved organ system is pulmonary
a classic orbital finding is bilateral painless enlargement of the lacrimal gland
the uveitis may be granulomatous or nongranulomatous
the anterior uveitis of sarcoidosis involves few or no synechiae
candle-wax drippings (taches de bougie) represent granulomas along retinal venules
Your answer was CORRECT
Explanation
Posterior and anterior synechiae can be extensive in sarcoidosis. Although the iridocyclitis in
sarcoidosis is classically granulomatous, it also can be non-granulomatous.
A 12-year-old boy presents with bilateral non-granulomatous uveitis. What kidney impairment is
he MOST likely to have?
IgA nephropathy
Fanconi syndrome
Goodpastures
tubulointerstitial nephritis
Your answer was CORRECT
Explanation
TINU (tubulointerstitial nephritis and uveitis) is an uncommon syndrome accounting for less than
2% of all cases of adult uveitis; however, it is a more common cause of uveitis in children and
adolescents, with a mean age of presentation of 15 years. Diagnosis is based on concurrence of
tubulointerstitial nephritis and bilateral acute anterior uveitis.
Goodpasture's syndrome is a rapidly progressive disease characterized by the onset of pulmonary
hemorrhage with glomerulonephritis, leading to pulmonary and renal failure due to anti-basement
membrane antibody. It is not associated with uveitis, though retinal and choroidal vasculitis can
occur.
This question came in the 2014 FRCOphth.
Classic Sjogren's syndrome consists of keratoconjunctivitis sicca, xerostomia, and:
SS-A and SS-B auto-antibodies
eczema
Hashimoto's thyroiditis
the Raynaud phenomenon
arthritis
Your answer was CORRECT
Explanation
The most frequent association is rheumatoid arthritis.
A patient with hepatic failure, dementia, and peripheral pigmentation of Descemet's membrane is
MOST likely to be diagnosed with:
Refsum's disease
incontinentia pigmenti
Fabry's disease
Wilson's disease
Alport's syndrome
Your answer was CORRECT
Explanation
These features suggest Wilson's disease.
Wilson's disease is autosomal recessive. It is caused by abnormalities in copper transport, with
low serum ceruloplasmin, paradoxically low serum copper, high urinary copper, and copper
deposition in tissues including the liver, brain and eyes. It results in liver failure and
neuropsychiatric symptoms. In the eyes, it can cause sunflower cataract from deposition of copper
within the anterior capsule (which is not usually visually significant) and Kayser-Fleischer rings
from deposition of copper in Descemet's membrane.